Questions Flashcards
Which of the following statements are true about substance use during pregnancy? Select all that apply.
A. Tobacco use during pregnancy can cause infants to have low birth weight.
B. Drinking even one beer per week puts Rose’s infant at risk of fetal alcohol syndrome.
C. Marijuana use during pregnancy causes characteristic facial abnormalities in the infant.
D. Cocaine and other stimulants such as amphetamines cause vasoconstriction that can lead to placental insufficiency and low birth weight.
> A, B, D have been selected by the expert.
Teratogens
A teratogen is an agent, or factor, that produces a malformation in the developing embryo. The agent may come from the external environment or may be a metabolite produced in excess by the mother or the fetus. Such agents include:
Drugs
Chemicals
Infections
Radiation
The effects of teratogens cannot be inherited.
Which of the following are components of the Apgar score? Select all that apply.
A. Color B. Heart rate C. Grimace D. Respiratory effort E. Capillary refill F. Tone
> A, B, C, D, F
A newborn receives a score of 0, 1, or 2 for each component, with the final Apgar score ranging from 0 to 10.
The score is reported at 1 minute and 5 minutes after birth for all infants.
The change in Apgar score between 1 and 5 minutes may be a useful indicator of response to resuscitation. According to Neonatal Resuscitation Program guidelines, a score below 7 at 5 minutes should prompt continued resuscitation, with re-assessment every 5 minutes, up to 20 minutes, until a score of 7 is achieved.
The Apgar score does not identify birth asphyxia and does not predict individual neurologic outcome or mortality.
What additional risks should you consider for a baby known to be SGA? Select all that apply.
A. Hypoglycemia
B. Hyperthermia
C. Hypothermia
D. Polycythemia (increased hemoglobin/hematocrit)
A, C, D
Which medications are routinely given to newborns and why? Select all that apply.
A. Intramuscular Vitamin K B. Oral Vitamin K C. Hepatitis B vaccine D. Hepatitis B Immunoglobulin E. Erythromcycin topical eye antibiotic
A, C, E
Vitamin K: Newborns routinely receive an intramuscular injection of vitamin K to prevent hemorrhagic disease of the newborn (also referred to as vitamin K deficiency bleeding, or, VKDB). The efficacy of oral Vitamin K is unknown.
Hepatitis B vaccine: One must consider maternal Hepatitis B status (HBsAg positive, status unknown or HBsAg negative) as well as the weight of the newborn (greater or less than 2000 grams) when deciding the timing of the first dose of Hepatitis B vaccine and whether or not Hepatitis B immunoglobulin is needed to decrease risk for vertical transmission.
Erythromycin (also tetracycline or silver nitrate): One of these antibiotics is administered topically to prevent gonococcal conjunctivitis.
A 33-year-old G1P0 female with a history of medically controlled seizures gives birth vaginally to a boy with IUGR at 38 weeks’ gestation. The newborn is noted to have dysmorphic cranial features and his head circumference is 28.5 cm (< 5th percentile). What is another associated abnormality you might expect to see in this newborn?
A. Hepatosplenomegaly B. Cardiac defects C. Absent red reflex D. Chorioretinitis E. Tremors
> B has been selected by the expert.
A. Hepatosplenomegaly in newborns is seen in metabolic diseases, storage diseases, HIV vertical transmission, intrinsic liver disease, and in congenital infections. Maternal anticonvulsant use does not cause hepatosplenomegaly.
B. The mother was on an anticonvulsant for her seizures. Taking anticonvulsants during pregnancy may lead to cardiac defects, dysmorphic craniofacial features, hypoplastic nails and distal phalanges, IUGR, and microcephaly. Mental retardation may be seen. A rare neonatal side effect is methemoglobinuria.
C. The red reflex is the normal reddish-orange reflection of light from the eye’s retina that is observed when using an ophthalmoscope. An absent red reflex in a newborn can be due to congenital cataracts or a retinoblastoma.
D. Chorioretinitis in a newborn may be due to congenital toxoplasmosis and CMV infections. It is not caused by maternal anticonvulsant use.
E. Tremors in a newborn can be due to maternal substance use. They would not result from maternal use of anticonvulsants.
A 19-year-old female in her 38th week of pregnancy goes into active labor. Shortly after birth her baby is noted to have a high-pitched cry, tremulousness, hypertonicity, and feeding difficulties. The baby is otherwise developmentally normal and the remainder of the physical exam also is normal. What is the drug the baby’s mother likely used during her pregnancy?
A. Heroin B. Alcohol C. Marijuana D. Cocaine E. Tobacco
> A has been selected by the expert.
A. Heroin is the correct choice. Opiate use during pregnancy may result in several different symptoms, including CNS findings (irritability, hyperactivity, hypertonicity, incessant high-pitched cry, tremors, seizures), GI symptoms (vomiting, diarrhea, weight loss, poor feeding, incessant hunger, excessive salivation), and respiratory findings (including nasal stuffiness, sneezing, and yawning).
B. Alcohol is incorrect. Fetal alcohol syndrome has a distinct pattern of facial abnormalities, growth deficiency, and CNS dysfunction. These infants may also exhibit other neurobehavioral deficits such as poor motor skills and hand-eye coordination and learning problems, such as difficulties with memory, attention, and judgment.
C. Marijuana is incorrect. There is limited evidence for a withdrawal syndrome associated with marijuana use.
D. Cocaine use during pregnancy is not typically associated with withdrawal symptoms. Cocaine has been linked to subtle deficits appreciated later in childhood, including deficits in cognitive performance, information processing, and attention to tasks.
E. Tobacco is incorrect. Smoking is not associated with the withdrawal syndrome described above. Smoking has been linked in a dose-dependent manner with lower weight newborns at birth. There is a two-fold increase in low birth weight even in light smokers (< 10 cigarettes per day). Smoking during pregnancy also has been associated with subtle neurodevelopmental deficits in some exposed children.
A 19-year-old G1P0 presents in labor to the ED at 38 gestational weeks. On interview it is discovered that the patient had irregular prenatal care, drank a couple of beers every weekend, and smoked 4 cigarettes a day. She delivers a baby boy who is small for gestational age. On exam, it is noted the baby has microcephaly, a smooth philtrum, and a thin upper lip. What do you suspect caused these features in the baby?
A. Tobacco exposure B. Alcohol exposure C. Congenital rubella D. Vertically transmitted HIV E. Congenital CMV infection
> B has been selected by the expert.
A. This choice is incorrect. While tobacco exposure can cause infants to be small for gestational age they typically do not have any characteristic facial features.
B. This choice is correct. Fetal alcohol syndrome has very characteristic facial features, including a smooth philtrum, thinning of the upper lip, and small palpebral fissures.
C. This choice is incorrect. Congenital rubella presents with sensorineural deafness, eye abnormalities (retinopathy, cataracts), and patent ductus arteriosus.
D. This choice is incorrect. Typically, vertically transmitted HIV does not lead to recognizable symptoms at birth. This diagnosis cannot be completely ruled out without lab testing.
E. This choice is incorrect. Symptomatic congenital CMV infection presents with microcephaly, jaundice, hepatosplenomegaly, low birth weight, and petechiae at birth.
A mother brings her 20-day-old male infant to your clinic for the child’s first visit. You learn that the infant was born at home to a 28-year-old G1P1, and the infant has not yet received newborn screening. During your history, you learn that the infant has been vomiting 2 to 3 times per day, and the mother reports that her son seems fussier than her friends’ infants. On exam, you note an eczematous rash and a musty odor to the infant’s skin and urine. Which enzyme deficiency would you expect the infant to display?
A. Phenylalanine hydroxylase B. Cystathionine synthase C. Sphingomyelinase D. Alpha-L-iduronidase E. Glucose-6-phosphatase
> A has been selected by the expert.
A. This infant likely has phenylketonuria (PKU), an autosomal recessive disorder of amino acid metabolism caused by a deficiency in the enzyme phenylalanine hydroxylase. Affected infants are normally detected by newborn screening, but can present with vomiting, hypotonia, musty odor, developmental delay, and decreased pigmentation of the hair and eyes. The best developmental outcomes occur if a phenylalanine-restricted diet is initiated in infancy.
B. A defect in cystathionine synthase occurs in homocystinuria, a disorder of amino acid metabolism. Homocystinuria is inherited in an autosomal recessive pattern. Individuals display Marfanoid body habitus, a hypercoaguable state, and possible developmental delay. The condition can be diagnosed by testing for increased methionine in a patient’s urine or blood.
C. A defect in sphingomyelinase occurs in Niemann-Pick disease, a lysosomal storage disease. Children present by six months of age with hepatomegaly, ataxia, seizures, and progressive neurologic degeneration. Fundoscopic exam reveals a “cherry-red” macula.
D. A defect in alpha-L-iduronidase occurs in Hurler syndrome, a type of autosomal recessive lysosomal storage disease. Children typically do not display symptoms until one year of age. Symptoms include hepatosplenomegaly, coarse facial features, frontal bossing, corneal clouding, and developmental delay. Affected individuals typically do not live past fifteen years old.
E. A defect in glucose-6-phosphatase occurs in Von Gierke’s disease, a glycogen storage disease. Von Gierke’s disease is inherited in an autosomal recessive pattern. Individuals present with hypoglycemia, hepatomegaly, and metabolic acidosis.
Of the following, which best reflects the caloric requirement of most healthy term babies in the first 1 to 2 months of life? Choose the single best answer.
A. 50 cal/kg/day
B. 100 cal/kg/day
C. 150 cal/kg/day
D. 200 cal/kg/day
B.
Term infant
Adequate growth for a term infant requires approximately 100 to 120 cal/kg/day. Average daily weight gain for a term infant is 20 to 30 grams.
Preterm infant
Preterm infants require 115 to 130 cal/kg/day.
VLBW (very low birth weight) infants
These infants require up to 150 cal/kg/day.
By what ages should an infant double and triple his or her birth weight? Choose the single best answer.
A. Double by 3 months, triple by 6 months
B. Double by 4 months, triple by 6 months
C. Double by 5 months, triple by 12 months
D. Double by 9 months, triple by 15 months
C.
Most healthy infants will double their birth weight by 4 to 5 months and will triple their birth weight by 1 year of age. In addition, most children will reach double their birth length by age 4 years.
Former preemies, small for gestational age babies, and others with chronic health issues do not always follow this pattern, but they are good general rules to expect in healthy term infants. Like many other standards in medicine, these are approximations.
Of the following, which are developmental milestones that you expect to see in a developmentally appropriate 9-month-old infant? Select all that apply.
A. Waves bye-bye B. Has a well developed pincer grasp C. Sits without support D. Walks well E. Says 2 words plus "mama" and "dada"
A and C
You are seeing a 36-month-old boy for his well-child visit. His parents are anxious about ensuring that his development is appropriate. He passed a hearing screen at birth and, other than a few colds, has been generally healthy. He has never been hospitalized or had any serious illness. He is able to run well, walk up stairs, and walk slowly down stairs. He uses more words than the parents are able to count, but can use them only in short, two or three-word sentences. His speech is understandable. He can draw a circle, but not a cross. Neurologic examination shows normal cranial nerves, normal sensitivity, normal motor reflexes, and no Babinski sign. Which of the following is the most appropriate next step in the management of this patient?
A. Perform a brain-stem auditory evoked potential hearing screen
B. Perform a screening exam for autism
C. Reassure the parents that the boy’s development appears normal
D. Refer the child to a developmental specialist for comprehensive evaluation
E. Refer the child to a specialist for evaluation of his delayed motor development
> C has been selected by the expert.
A. A brain-stem auditory evoked potential hearing test (BAER) may be indicated in infants who fail to meet language milestones if they cannot cooperate with other more comprehensive testing. A 36-month-old should be able to cooperate with behavioral audiometry, so a BAER is not indicated. In addition, this child has no evidence of language delay and does not require referral at this point.
B. Autism is an increasingly diagnosed cause of developmental delay, but this child is not delayed and no mention is given of any autistic features, such as a lack of symbolic play, repetitive movements, or poor sociability.
C. The developmental milestones mentioned in the vignette are within the range of normal for a 36-month-old child. In the absence of any other evidence of significant impairment, there is no indication for referral at this point.
D. If there are reasons for concern on developmental screening tests, a referral may be indicated. However, the developmental milestones mentioned in the vignette are within the range of normal for a 36-month-old child.
E. This child’s motor milestones are not delayed, and no referral is indicated.
Sammy is a healthy male child brought into your office by his mother for a well-child examination. As part of your evaluation you assess his developmental milestones. He is able to run, make a tower of 2 cubes, has 6 words in his vocabulary, and can remove his own garments. What would you estimate Sammy’s age to be based upon his developmental milestones?
A. 12 months B. 15 months C. 18 months D. 30 months E. 36 months
> C has been selected by the expert.
A. At age one year, gross motor skills include pulling to stand, standing alone, and perhaps first steps. Fine motor skills including putting a block in a cup and banging 2 cubes held in hands. At this age a child should be able to imitate vocalizations/sounds and babble. The majority of children this age will know 1 or 2 words in addition to “mama” and “dada.” Social-emotional milestones at age one year are waving bye-bye and playing pat-a-cake. Running, building towers of blocks, removing clothing, and a 6-word vocabulary are more advanced skills than a 12-month-old would be expected to have.
B. At 15 months of age, a child should be able to stoop and recover and walk well, put a block in a cup, have a vocabulary of a few words, wave bye-bye, and drink from a cup. Running, building towers of blocks, removing clothing, and a 6-word vocabulary are more advanced skills than a 15-month-old would be expected to have.
C. At 18 months, a child should be able to walk backward, and 50-90% of children can run at this stage. An 18-month-old should be able to scribble, build a tower of 2 cubes, have 3-6 words in her or his vocabulary, and be able to help in the house and remove garments.
D. At 2 ½ years of age, kids can jump up and throw a ball overhand. They can build a tower of 6-8 cubes, point to 6 body parts, name 1 picture, put on clothing, and wash and dry their hands. Sammy is only able to build a tower of 2 cubes, can remove his clothing but does not yet put clothing on, and his vocabulary is limited to 6 words-leading us to believe he is not 2 ½ years old.
E. At age 3, children can balance on each foot for 1 second, wiggle their thumbs, name 4 pictures, name 1 color, name a friend, and brush their teeth with help. Sammy’s vocabulary is only 6 words, he is not able to name a friend, he is only able to stack 2 cubes, and he has just starting running, but is unable to balance on each foot for 1 second.
Mark is a 5-month-old male who is brought to the urgent care clinic with a three-day history of rhinorrhea and non-productive cough. When he was born he was large for gestational age, and his exam then was notable for macrocephaly, macroglossia, and hypospadias. On physical exam now his vitals signs are stable. He has copious nasal discharge, but his lungs are clear to auscultation. On abdominal exam, you palpate an abdominal mass on the right side just below the subcostal margin. It is 7 cm in diameter and does not cross the midline. The abdomen is soft and non-tender with active bowel sounds. What is the most likely cause of his mass?
A. Wilms’ tumor
B. Teratoma
C. Renal cell carcinoma
D. Hepatoblastoma
> A has been selected by the expert.
A. Wilms’ tumor is commonly associated with Beckwith-Wiedemann syndrome, a genetic overgrowth syndrome. Other features that may be seen in children with this syndrome include omphalocele, hemihypertrophy, hypoglycemia, large for gestational age, and other dysmorphic features.
B. Teratomas are congenital tumors that are present at birth. These benign tumors that are often identified incidentally, or may become symptomatic due to mass effect of the lesion within the abdominal cavity. The aggressiveness of the tumor depends on the degree of differentiation.
C. Renal cell carcinomas are much more common in adulthood. Risk factors include cigarette smoking and obesity.
D. While children with Beckwith-Wiedemann syndrome can have hepatoblastoma (in addition to other types of tumors), this is not the most common tumor in this genetic condition. Note that hepatoblastoma may also be associated with familial adenomatous polyposis.
An asymptomatic, healthy 9-month-old female is found to have a palpable RUQ mass on exam. After further imaging and lab studies, the mass is diagnosed as a neuroblastoma that has involvement in the bone marrow as well. The mother is worried about the prognosis. Which of the following is true about the prognosis of neuroblastoma in this child?
A. Lymph node involvement is a poor prognostic factor
B. Prognosis of neuroblastoma is predictable
C. Children who are older than 12 months have a better prognosis than younger children
D. Favorable histology does not play a role in prognosis
E. Non-amplification of the n-myc gene is a favorable prognostic factor.
> E has been selected by the expert.
A. Due to the effectiveness of chemotherapy, neuroblastomas with lymph node involvement are still considered favorable, especially in the setting of other favorable factors, such as young age and differentiating histology. Though distant metastasis is a significant poor prognostic factor, regional lymph nodes do not significantly affect the outcome.
B. Neuroblastoma has a broad spectrum of clinical courses. Some tumors may spontaneously regress, some may mature to a benign type, and yet other tumors can be very aggressive with metastases. Age plays a role in the prognosis, as most infants have a good prognosis even with disseminated disease, while infants over 18 months of age do not do as well.
C. In infants less than one year of age, neuroblastoma tumors may spontaneously regress. Stage 4S neuroblastoma is a special category that is reserved for infants less than 12 months who have resectable primary tumors and metastases to the liver, skin, and bone marrow. Overall survival is over 85 percent for babies over 6 weeks of age with Stage 4S.
D. Favorable histology is a good prognostic factor in neuroblastoma, and is based on the differentiation of the cells involved.
E. Non-amplification of the n-myc gene is one of the favorable genetics in neuroblastoma.
A 9-month old baby boy comes to the clinic for a well child visit. The child is at the 50th percentile for weight, length, and head circumference. He is reaching all developmental milestones appropriately. The mother has no concerns at this visit. The child has previously received the following vaccines: 3 doses of DTaP, 3 doses of Hib, 2 doses of HepB, 3 doses of RotaV, 2 doses of IPV and 3 doses of PCV13, and no influenza vaccines. Which vaccines should the child receive at today’s visit?
A. Influenza, Hep B, IPV, DTaP B. Influenza, IPV C. Influenza, Hep B, IPV D. Hep B, DTaP, IPV E. Hep B, IPV, and MMR
> C has been selected by the expert.
A. Influenza, Hep B, IPV, DTaP is incorrect. All three doses of DTaP have been given.
B. Influenza, IPV is incorrect. The patient needs the third Hep B shot.
C. Influenza, Hep B, IPV is correct. The patient needs a third Hep B, a third IPV, and a yearly flu shot starting at 6 months of age.
D. Hep B, DTaP, IPV is incorrect. All three doses of DTaP have been given and the patient now needs a yearly flu shot starting at 6 months of age.
E. Hep B, IPV, and MMR is incorrect. The patient also needs a yearly flu shot starting at 6 months of age and MMR is not given before 12 months of age.
A 10-month-old asymptomatic infant presents with a RUQ mass. Work-up reveals a normocytic anemia, elevated urinary HVA/VMA, and a large heterogeneous mass with scant calcifications on CT. A bone marrow biopsy is performed. Which of the following histologic findings on bone marrow biopsy is most consistent with your suspected diagnosis?
A. Sheets of lymphocytes with interspersed macrophages
B. Small round blue cells with dense nuclei forming small rosettes
C. Hypersegmented neutrophils
D. Stacks of RBCs
E. Enlarged cells with intranuclear inclusion bodies
> B has been selected by the expert.
A. This is incorrect, as sheets of lymphocytes with interspersed macrophages are associated with Burkitt lymphoma.
B. This is the correct response. In addition to neuroblastoma, other tumors associated with small blue cells include Ewing’s sarcoma and medulloblastoma, both of which tumors are seen in children.
C. Hypersegmented neutrophils are characteristic of megaloblastic anemia, a condition associated with a vitamin B12 and/or folate deficiency, not malignancy.
D. Stacks of RBCs suggest rouleaux formation, a phenomenon seen in multiple myeloma, a condition not seen in young infants.
E. This describes the classic “owl’s eyes” seen in CMV and other viral infections.
Which of the following safety issues are important to discuss with the parents of a 3-year-old? Select all that apply.
A. Rear-facing car seat B. SIDS C. Car accidents D. Swimming-pool safety E. Falls F. Firearms in the home G. Poisonings H. Fire safety
C, D, E, F, G, H
A 3-year-old boy presents for a follow-up visit after being diagnosed with iron deficiency anemia. He is currently receiving oral iron supplements, 2 mg/kg of elemental iron daily. He has a dietary history of eating mostly sweet, bland, low-texture foods. What strategies may be used to improve his diet?
A. Continue bottle-feeding
B. Encourage eating small amounts of food throughout the day (grazing)
C. Gradually introduce new foods and slowly decrease his old favorites
D. Bargain and cajole with the child
E. No change is needed; bland, low-texture foods are optimal for a child this age
> C has been selected by the expert.
A. This choice is incorrect because the child should stop bottle-feeding now to improve his diet. Children usually stop requesting the bottle a few days after it has been discarded.
B. This choice is incorrect because the child should be encouraged to restrict eating to 3 meals and 2 snacks per day, instead of “grazing” throughout the day.
C. This choice is correct, because gradually introducing new foods and slowly decreasing his old foods will likely ease the transition to healthier diet choices and encourage long-term adjustment.
D. This choice is incorrect because bargaining and cajoling is unlikely to be effective. He should be presented with healthy options only, and dessert should not be used as an incentive for healthy eating.
E. This choice is incorrect because the child’s current diet is not optimal. He should be eating a varied diet with the recommended servings of fruits and vegetables per day.
A 2-year-old girl is examined as an outpatient. While waiting for the pediatrician, her mother reads her a short book. When they are done, her mother asks her to take the book and return it to a bookshelf in the room. The child is not able to hold a pencil and cannot write her name. She can kick and throw a ball, but cannot jump in place. Which of the following best describes this child’s development?
A. Delayed language B. Delayed social skills C. Advanced fine motor skills D. Advanced gross motor skills E. Age-appropriate development
> E has been selected by the expert.
A. Delayed language is incorrect: A 24-month-old child is expected to use pronouns inappropriately, but should be able to follow two-step commands such as taking a book and returning it to a location in the room.
B. Delayed social skills is incorrect: At 24 months of age, children are able not only to listen to short stories, they also engage in parallel play.
C. Advanced fine motor skills is incorrect: While a child can hold a pencil at 24 months, the grip is immature and the child imitates pencil strokes. Children can remove their pants and socks at this age, but need help to undress completely.
D. Advanced gross motor is incorrect: Being able to jump in place is a 30-month-old milestone. Being able to throw a ball overhand is expected at 24 months of age.
E. The child in this vignette is developmentally appropriate for her age.
At a routine well-child visit, the frantic mother of your 4-year-old male patient states that she thinks her son has some developmental delays based on what she hears from other parents. Although he knows how to do such things as throw a ball and copy a circle, he cannot brush his teeth on his own, tie his shoes, or hop on one foot. According to the AAP’s Bright Futures, which of the following are development milestones for typical 4-year-olds?
A. Throw a ball overhand, ride tricycle, build tower of 6-8 cubes
B. Hop on 1 foot, copy a cross, brush teeth
C. Tie a knot, copy squares
D. Mature pencil grasp, print some letters and numbers
E. Skip, draw a person with 6 or more body parts
> B has been selected by the expert.
A. This choice is incorrect. Throwing a ball overhand, riding a tricycle, building a tower of 6-8 cubes, and copying a circle are developmental milestones for 3-year-olds.
B. This choice is correct. A a normally developing 4-year-old should be able to hop on 1 foot, copy a cross, pour/cut/mash their own food, and brush teeth.
C. This choice is incorrect. These are milestones for 5-year-olds.
D. This choice is incorrect. These are milestones for 5-year-olds.
E. This choice is incorrect. These are milestones for 5-year-olds.
A 3-year-old boy described by his mother as a picky eater comes in for a regularly scheduled well-child visit. His mother complains that he has had less energy than usual for the past few months. There is a high clinical suspicion he is anemic. Which of the following is most correct?
A. The most cost-effective test to diagnose anemia is a CBC.
B. Lead screening is never warranted since a 3-year-old is usually not mouthing objects.
C. The most likely cause of anemia in the question is picky eating resulting in low iron intake, which would cause microcytic anemia.
D. The most common cause of anemia in this situation is folate deficiency.
E. If anemia is due to poor nutrition, restarting the bottle will help the child recover the most.
> C has been selected by the expert.
A. This choice is incorrect. The most cost-effective method to diagnose anemia is a screening hemoglobin.
B. This choice is incorrect. Although most 3-year-olds stop mouthing objects, they may have been exposed to lead in the past but were not symptomatic at the time.
C. This is the correct choice. The most likely cause of anemia in the question is picky eating, which can result in insufficient iron intake. Low iron intake causes a microcytic anemia. A girl with menometrorrhagia would present with iron deficiency, and her MCV also would be indicative of microcytic anemia.
D. This choice is incorrect. Iron deficiency is the most common cause of anemia in this scenario. Note that folate deficiency may be associated with a goat’s milk diet.
E. This choice is incorrect. It is best to stop the bottle by age one year. Solid foods provide more complete nutrition, including iron. An additional concern of prolonged bottle usage is the development of dental caries.
A 5-year-old girl comes into your office for a well-child visit. The mother says that child is overall very healthy, but she highlights “occasional colds” and recently more frequent temper tantrums. She does well in preschool, is toilet trained, and enjoys eating mostly pasta, bread, and milk. She lives with her mother and father in a home built in 1985. Lab studies were significant for a mild anemia with a hemoglobin of 10.0 g/dL. You note that her hemoglobin was in the normal range at her 3-year-old visit. Which of the following is the most likely cause of her anemia?
A. Chronic blood loss B. Lead poisoning C. Chronic illness D. Iron deficiency E. Hemoglobinopathy
> D has been selected by the expert.
A. This choice is incorrect. There was no complaint of melena, and the child is overall healthy. Chronic blood loss would therefore be unlikely.
B. This choice is incorrect. Risk of lead poisoning is increased in patients who live in homes built before the 1950s. Additionally, this patient does not complain of other symptoms suggestive of lead poisoning: weight loss, lethargy, vomiting, and learning difficulties.
C. This choice is incorrect. The patient has no significant past medical history, and the review of systems is unremarkable. Chronic illness is unlikely.
D. This choice is correct. Given the patient’s age and preference for pasta and milk, the most likely cause of anemia would be iron deficiency. Treatment would include oral iron supplementation and increased dietary iron intake.
E. This choice is incorrect. If she had a hemoglobinopathy, she would have been expected to have been anemic at her previous visit as well.
Which of the following statements regarding stimulant medications are true? Select all that apply.
A. Stimulant medications are addictive when used to treat children with ADHD..
B. Prolonged use of stimulant medication is associated with later increased incidence of substance abuse.
C. Stimulant medications may decrease appetite.
D. Stimulant medications simply mask behavioral problems by sedating the child.
E. Stimulant medications can cause tics.
F. Patients may develop insomnia.
G. Stimulant medication may cause decreased growth velocity.
H. Stimulant medication leads to an increased risk of sudden cardiac death in otherwise healthy children.
C, F, G
Which of the following are risk factors for childhood obesity? Select all that apply.
A. Breastfeeding B. High birth weight C. Obese parent D. Delayed puberty E. Lower socioeconomic status F. Prader-Willi syndrome
> B, C, E, F
Prenatal/neonatal risk factors for obesity include high birth weight and maternal diabetes.
Having an obese parent increases a child’s risk for adult obesity, and the risk increases significantly if both parents are obese:
Odds ratio for one parent is 3
Odds ratio for both parents is more than 10
Before age 3 years, parental obesity is a stronger predictor of obesity in adulthood than child’s weight status.
Children from families of lower socioeconomic status have higher rates of obesity. This is likely due to multiple factors, including lack of safe places for physical activity and less access to nutrient rich, healthy foods and beverages.
Certain genetic syndromes (such as Prader-Willi, Bardet-Biedl, and Cohen syndromes) are known to be associated with obesity.
The remaining answer choices are not correct:
In infants, the extent and duration of breastfeeding (A) is inversely associated with obesity risk in childhood. This may be due to physiologic factors in human milk as well as the feeding and parenting patterns associated with nursing.
Early menarche, not delayed puberty (D), has been associated with a BMI greater than the 85th percentile.
Which of the following are possible sequelae of obesity? Select all that apply.
A. Obstructive sleep apnea B. Dyslipidemia C. Reactive airway disease D. Hypertension E. Slipped capital femoral epiphysis F. Type I diabetes mellitus G. Type II diabetes mellitus H. Steatohepatitis
> A, B, D, E, G, H have been selected by the expert.
Morbid obesity may directly cause restrictive lung disease, but is not currently thought to be causally linked with asthma/reactive airway disease (RAD) (C). Poorly controlled RAD, which in turn impairs exercise tolerance, may contribute to obesity. There is an association of increased asthma in patients with obesity, but the relationship is currently thought of as an association that requires further research, but not causal.
Which of the following may elevate blood pressure measurements? Select all that apply.
A. Anxiety over being at the doctor’s office
B. Use of a BP cuff that is too large
C. Taking a BP below the level of the heart
D. Patient is in pain
A, C, D
“White coat” hypertension due to the anxiety of being at the doctor’s office is a common cause of elevated BP measurements.
Taking several BP readings in succession may reveal a steady decline in the BP toward the normal range in such cases.
A school nurse may be a valuable asset in obtaining serial BP readings over time in a less-threatening environment.
Positioning may influence BP readings.
The patient should be seated, in a relaxed state, with the arm held at the level of the heart.
Holding the arm down at the side may elevate the systolic BP as much as 20mmHg to 30 mmHg in an adolescent.
Studies have shown that painful stimuli can acutely raise BP.
Remember that a cuff that is too large will give a falsely low BP measurement; conversely, a cuff that is too small may give a falsely elevated BP.
A BP cuff should cover 2/3 of the upper arm.
The internal bladder should encircle 80% to 100% of the arm circumference.
Although obesity is commonly associated with hypertension, it is also common that obese children have their BP checked with a cuff that is theoretically appropriate for their age but inappropriately small for their size.
An 8-year-old boy is brought to clinic by his parents because they are concerned that he has not been doing his homework. His teacher recently called the parents to say that their son seems distracted in class, constantly interrupts other children when they are speaking, and is very fidgety. When you speak with the boy, he tells you that he did not know about the homework assignments and that he tries hard to pay attention in class. What is the next best step in management?
A. Prescribe a stimulant medication for ADHD
B. Suggest behavior modification for the child and parenting classes
C. Group therapy for the child
D. Do nothing, as this child’s behavior is normal
E. Contact the teacher to find out more about his behavior. Find out more about the child’s behavior at home
> E
A. Pharmacotherapy is often used in combination with behavioral modification/group therapy for children diagnosed with ADHD. However, to be diagnosed with ADHD, one must have 6 or more symptoms in 2 or more settings for at least 6 months, and several of these symptoms must be present before the age of 12. The symptoms fall within the three categories of inattention, hyperactivity, and impulsivity.
B. Behavior modification for the child and parenting classes for the parents are both used as treatment modalities in ADHD. Parents should be counseled on positive reinforcement, firm non-punitive limit setting, and how to reduce external stimuli. However, this child first requires further evaluation.
C. Children with ADHD often learn best in group therapy, particularly social skills and self-esteem. Again, this child first requires further evaluation.
D. Many school-aged children are easily distractible, impatient, and hyperactive. It is important to distinguish those who truly have ADHD from those who do not. Again, to have a diagnosis of ADHD, symptoms must be present in two or more settings.
E. Contacting the teacher to find out more about the child’s behavior at school and learning more about his behavior at home are the best ways to determine if 6 of the symptoms are present in 2 or more settings, which is required to make the diagnosis of ADHD. It also will be important to learn more about other aspects of this child’s life, as there are several factors that can lead to acting out (including learning disability, hearing disability, family stress, and abuse).
An 8-year-old healthy obese African American male with no past medical history is found to have a blood pressure of 125/90 mmHg on all four extremities on routine evaluation during an office visit for well-child care. Review of symptoms is negative. A physical exam and screening bloodwork are performed. Both are normal, with the exception of his blood pressure and obesity. What is the most likely diagnosis?
A. Primary hypertension B. Renal artery stenosis C. Coarctation of the aorta D. Pheochromocytoma E. Hyperthyroidism
> A has been selected by the expert.
A. The sole physical finding is hypertension. Given the mild hypertension and the patient’s age, symptoms are unlikely to be present. Other etiologies should be ruled out, but review of symptoms, physical examination, and laboratory studies do not suggest other etiologies.
B. Patients with renal artery stenosis are largely asymptomatic, but as the kidney function deteriorates, they may experience edema and dyspnea. On physical examination, patients will have hypertension (as a consequence of elevated angiotensin II and aldosterone) and possibly abdominal bruits. For laboratory studies, the patient will have elevated serum angiotensin II and serum aldosterone, as the kidneys attempt to compensate for a perceived decrease in glomerular filtration rate.
C. Assuming sufficient severity of the coarctation, symptoms include chest pain, cold lower extremities, dizziness, syncope, exercise intolerance, failure to thrive, poor growth, headache, and dyspnea, among others. Distal to the coarctation, pulses will be diminished. Blood pressure will be lower in the lower extremities compared to the upper extremities. Murmurs may also be heart on auscultation. Coarctation of the aorta may also be associated with other congenital heart defects (bicuspid aortic valve in 50% of patients) and chromosomal abnormalities (Turner syndrome).
D. Symptoms include headache, diaphoresis, palpitations, tremor, nausea, weakness, anxiety, nervousness, irritability, and weight loss, among other symptoms. On physical examination, patients typically present with tachycardia and severely elevated blood pressure. For laboratory studies, the patient will have elevated urinary VMA, urinary HVA, urinary metanephrines, and serum glucose. Pheochromocytoma may be associated with various syndromes, such as multiple endocrine neoplasia and Von Hippel-Lindau disease.
E. Symptoms include heat intolerance, frequent bowel movements, increased appetite, diaphoresis, nervousness, restlessness, weight loss, tremor, hair loss, and palpitations, among others. On examination, patients typically present with tachycardia, hypertension, hyperreflexia, and goiter. The patient will have elevated T3, T4, and serum glucose, and TSH may be depressed or elevated depending on the etiology.
Billy, a 7-year-old boy, presents to the clinic with complaints of headaches and episodes of feeling sweaty and flushed. He also reports that at times he feels as if his heart is racing. Billy was full term, had an uncomplicated birth, and has been otherwise healthy until now. On exam his BP is 120/80 mmHg and is the same in his upper and lower extremities. His weight and height are in the 50th percentile for his age. What is a likely cause of Billy’s hypertension?
A. Coarctation of the aorta B. Renal vascular disease C. Renal insufficiency due to renal scarring D. Catecholamine excess E. Primary hypertension
> D has been selected by the expert.
A. Coarctation of the aorta should be suspected in a child with elevated BP (usually > 99th percentile), little family history of HTN, and a discrepancy between upper and lower extremity BPs. Some children with coarctation of the aorta may go undetected until presenting with hypertension at a school-age visit. It is important to pay special attention to the femoral pulses and to document BP measurement in a lower extremity. Billy’s BPs in his upper and lower extremities are the same, making this diagnosis less likely.
B. Umbilical arterial or venous lines as neonate (most often in premies) can predispose a child to renal vascular disease. Billy had an uncomplicated birth and did not go the neonatal ICU, making it less likely that his hypertension is due to renal vascular disease secondary to an umbilical arterial or venous line as a neonate.
C. Recurrent urinary tract infections in childhood are one of the leading causes of hypertension and renal insufficiency later in life due to renal scarring following infections. UTIs are more common in girls. Billy does not have a stated history of urinary tract infections, and his palpitations and flushing are not consistent with hypertension secondary to UTIs.
D. Catecholamine excess (pheochromocytoma or neuroblastoma) should be suspected in a child who is hypertensive and has episodes of sudden sweating, flushing, or feels that his heart is racing. Billy is exhibiting these signs and a urine catecholamine testing would be appropriate in this case.
E. Most hypertension in children over 6 years of age, and in adolescents, is due to primary HTN. Obesity is an important correlate. Billy’s flushing and racing heart would not be fully explained by primary hypertension. Also, Billy is an appropriate weight, making this diagnosis less likely.
Jane is an 8-year-old girl who presents to your clinic for follow-up after being hospitalized for status asthmaticus. She has just completed a 10-day course of systemic steroids. Given her history of moderate persistent asthma, her outpatient regimen includes Advair, a combined steroid and bronchodilator. She was also diagnosed with ADHD one year ago and was started on Concerta, 18 gm PO once a day. Her BMI today is at the 83rd percentile for her age, and her blood pressure is at the 98th percentile for her age. What is the most likely cause of her stage I hypertension?
A. Obesity B. The blood pressure cuff is too big C. Medications D. Renal insufficiency E. Neurofibromatosis 1
> C has been selected by the expert.
A. While obesity is a risk factor for hypertension, Jane is not overweight. A child is considered overweight when his or her BMI is between the 85th and 95th percentile for age. Obesity is considered > 95th percentile.
B. Blood pressure would be falsely decreased if the cuff was too big, and, inversely, falsely elevated if the BP cuff was too small.
C. Both steroids and amphetamines can cause increases in blood pressure, especially when used in combination. Steroids increase blood pressure by mimicking endogenous cortisol and the sympathetic fight or flight response. Amphetamines mimic norepinephrine, stimulating alpha and beta adrenergic receptors, causing an overall increase in blood pressure.
D. Renal insufficiency can be a cause of secondary hypertension, but is unlikely in this patient. Risk factors that warrant investigation of renal causes for hypertension include recurrent UTIs, umbilical arterial/venous lines placed while a child was in the ICU, and a family history of renal disease.
E. NF-1 can be associated with hypertension as a result of vascular malformations that affect blood supply to the kidneys; however, this child does not present with any signs of NF-1 (café au lait macules, neurofibromas, optic gliomas, lisch nodules).
George is a 7-year-old boy frequently in trouble at school for being disruptive and inappropriately talkative in class, not following directions set by his teacher, and not working well with classmates during group activities. His mother relates that at home George is always on the go, sleeping only 6 to 7 hours a night. He does not follow her rules all the time either, including not doing his homework, and sometimes putting himself in danger by doing things she tells him not to do, such as running away unaccompanied. Which of the following is the most likely diagnosis?
A. Bipolar mood disorder B. Anti-social personality disorder C. Conversion disorder D. ADHD E. Rett syndrome
> D has been selected by the expert.
A. Depression may be responsible for the inattention this child exhibits in school. It is not uncommon for childhood depression to lead to bipolar disorder, in which hyperactivity and impulsivity comprise the manic phase of the disorder leading to a decreased need for sleep. However, the symptoms of depression and mania present in separate phases, not concurrently. The mnemonic commonly used for mania is DIGFAST (distractability, irresponsibility, grandiosity, flight of ideas, agitation, sleep decrease, talkativeness). The mnemonic for depression is SIGECAPS (sleep disturbance, loss of interest, guilt, energy loss, concentration impairment, appetite changes, psychomotor retardation, suicidal ideation).
B. Symptoms of anti-social personality disorder include inability to conform to societal norms, disregard of the rights of others, and often criminality. These individuals often exhibit impulsiveness due to their lack of consideration of the consequences of their actions. Males are affected more than females. However, a diagnosis of anti-social personality disorder can be made only in individuals older than 15 years, earlier than which a diagnosis of conduct disorder is appropriate.
C. Conversion disorder is a subtype of somatoform disorder. Somatoform disorder would be suspected if no cause could be identified for reported physical symptoms. The drive is unconscious on the part of the patient, and symptoms are not intentionally produced or faked. Conversion disorder is more common in adolescents and involves a sudden loss of sensory or motor functioning. When the patient consciously creates physical symptoms this is referred to as factitious disorder.
D. ADHD is characterized by the triad of impulsivity, hyperactivity, and inattention. Other symptoms include motor impairment and emotional labiality. ADHD is typically diagnosed before the age of 7 but persists into adulthood. Intelligence is usually normal, but individuals with ADHD commonly perform more poorly academically than would be expected for their IQ.
E. Rett syndrome is an X-linked pervasive developmental disorder seen only in females; affected males die in utero or at birth. The characteristic symptoms involve regression of language and development, intellectual disability, ataxia, and hand-wringing. This disorder is typically diagnosed earlier, at about age 1-4.
What is the most common cause of chest pain in an adolescent?
A. Asthma B. Gastroesophageal reflux disease (GERD) C. Musculoskeletal D. Myocardial ischemia E. Pericarditis
> C has been selected by the expert.
Musculoskeletal issues are by far the most common cause of chest pain in children.
Asthma and GERD are potential causes of chest pain, but are unlikely without other associated symptoms.
Unlike adults, chest pain due to a cardiac cause is extremely uncommon in children.
TEACHING POINT
Distinguishing Types of Chest Pain
Precordial catch syndrome is the most common cause of chest pain in adolescents and is of unknown etiology. It is a benign condition characterized by sudden, sporadic onset of sharp pain, usually along the left sternal border, which is often exacerbated with deep inspiration. These pains are brief, lasting seconds to a few minutes, and resolve spontaneously. The pain can often be “broken” with a forced deep inspiration. It is often not associated with exercise.
Costochondritis is also a benign cause of chest pain but less common. The pain of costochondritis is due to inflammation and typically will last for hours or days.
Gastrointestinal causes of chest pain include gastroesophageal reflux, gastritis, and esophagitis. These often cause pain that is described as:
Retrosternal
Burning
Non-radiating
Associated with meals
A medication history is important in discerning a possible GI cause of chest pain and should include non-prescription meds, oral contraceptives, and those causing gastric irritation. Social history should inquire about the use of the following, all of which might potentially cause gastric or esophageal irritation:
Alcohol
Tobacco
Intoxicants
Stimulants
Cocaine or other drugs
Asthma or exercise-induced bronchospasm (EIB) may cause chest pain - usually in association with cough, wheezing, or respiratory distress.
Important reasons to perform a GU exam during a pre-participation sports evaluation include which of the following?
A. Tanner staging for selecting sports
B. Hernia check
C. Check for undescended testicles
D. Demonstrate testicular self-exam
B, C, D
Which of the following immunizations are routinely administered at the 11-year-old well-child visit? Select all that apply.
A. HPV Human Papilloma Virus
B. Meningococcal B
C. Meningococcal ACYW Conjugate vaccine
D. DTaP (tetanus/diphtheria/acellular pertussis)
E. Tdap (tetanus/diphtheria/acellular pertussis)
A, C, E
John is a 17-year-old presenting today for a pre-participation physical exam. During the interview, he reports a low-grade fever, malaise, and headache for one week. In the past few days, his fever has gotten worse and he complains of a sore throat. He denies cough or chest pain. On physical examination, he is found to have a temperature of 101.3° F, and cervical lymphadenopathy and oropharyngeal erythema with exudate are noted. His participation would be most likely affected by which of the following tests?
A. Chest x-ray B. CT head/neck C. EBV serologies D. Throat culture E. No further workup
> C has been selected by the expert.
A. This choice is incorrect because there is no evidence of lower respiratory tract signs or symptoms.
B. This choice is incorrect because the additional exposure to radiation is not warranted given the availability of other higher priority tests.
C. This choice is correct because the patient’s symptoms are suggestive of infectious mononucleosis. These include complaints of low-grade fever and malaise and findings of cervical lymphadenopathy and pharyngeal exudate. If testing is positive, the patient should be restricted from strenuous activity or contact sports during his illness due to the risk of splenic rupture.
D. An antigen test (and culture, if negative) should be performed for streptococcal pharyngitis, but this clinical picture is more consistent with EBV, and mononucleosis is more likely to preclude participation in sports. After 24 hours of antimicrobials, the patient with streptococcal pharyngitis is no longer contagious.
E. This choice is incorrect because the patient’s symptoms and findings warrant further evaluation.
A 17-year-old boy presents for a sports pre-participation physical. He reports that he occasionally gets short of breath and feels light-headed with exercise, and sometimes he experiences chest pain as well. He lost consciousness once last season during a playoff basketball game, but attributed it to feeling sick at the time. His grandfather died suddenly at age 35 of unknown etiology. Which of the following is the most likely diagnosis?
A. Hypoglycemia B. Congenital heart block C. Postural hypotension D. Prolonged QT syndrome E. Ventricular septal defect
> D has been selected by the expert.
A. Hypoglycemia is very uncommon in healthy children. It is usually associated with diaphoresis, anxiety, tremulousness, and a feeling of hunger. Chest pain and shortness of breath are less likely.
B. Congenital heart block can be caused by congenital heart defects and autoimmune disease in pregnant women (lupus). This condition is a rare cause of arrhythmia and is often diagnosed early in life. Many patients require a pacemaker.
C. Postural hypotension is a very common cause of dizziness and visual changes in the pediatric population. It is usually triggered by volume depletion and skipping meals. Hypotension is not usually associated with shortness of breath, chest pain, and prolonged (> 5 min) loss of consciousness.
D. Prolonged QT syndrome can cause syncopal episodes in late childhood or adolescence. QT intervals are elongated on ECG and lead to arrhythmias, like ventricular fibrillation. This condition is often associated with other abnormalities, including severe congenital sensorineural deafness.
E. Ventricular septal defects often have negligible cardiovascular sequelae, or will close up on their own after a number of years. If they are not monitored, however, the shunting can cause hypertrophy of the right heart, called Eisenmeiger syndrome, which could result in pulmonary hypertension, exercise intolerance, and eventual heart failure. It would be unlikely, however, to cause chest pain or syncope. The family history of sudden cardiac death also points to prolonged QT rather than a VSD.
A 16-year-old male presents to your office requesting clearance to play football. You begin by taking his medical history. He says that he feels very well, but admits that he recently experienced one episode of syncope that occurred when he trained really hard for football tryouts with his friends. He denies any shortness of breath, or chest pain currently. Family history is significant for an uncle who died of heat stroke at the age of 30 while playing basketball. Physical examination reveals no abnormalities. What is the next best step in management?
A. ECG now, and if normal, reassurance B. Medically clear him to play C. Stress test D. ECG and referral to cardiology E. Observe and follow up in 6 months
> D has been selected by the expert.
A. An ECG is indicated in all cases of syncope. Neurocardiogenic syncope, also known as vasovagal syncope, occurs when the brain is not being adequately perfused. Predisposing factors may include dehydration, hypovolemia, and standing up too quickly. It is not usually dangerous, and in these cases an ECG would be an appropriate first step. However, in cases of atypical syncope, as above, in which syncope occurs with exertion, and ECG would not be a sufficient work up.
B. All medical issues must be resolved prior to clearing a patient to participate in a sport. Syncope with exertion, during exercise, is very concerning, and this patient should be worked up appropriately prior to clearance.
C. A stress test is appropriate for evaluating chest pain, particularly in those at risk for atherosclerosis. However, in a patient with known syncope with exertion, a stress test might actually be dangerous. In addition, it only considers vessel disease as a source of pain, and does not take into account structural issues like an outlet obstruction.
D. Referral to cardiology is the absolute next best step! The combination of syncope with exertion and a family history of a young death is concerning for something like hypertrophic cardiomyopathy. Don’t be fooled about heat stroke. That is a positive family history for sudden death in a young person. This patient must be evaluated by cardiology, even if you don’t hear a cardiac murmur!
E. Observation is not appropriate here. As mentioned above, the syncopal event and positive family history are concerning. It would be inappropriate to just observe if this patient has hypertrophic cardiomyopathy, a very significant risk factor for sudden death. Cardiology consult is the only appropriate option here.
A 16-year-old previously healthy male comes to the Pediatrics Urgent Care Clinic having “almost fainted” at soccer practice. He says that he had not eaten much earlier in the day and it was very hot and muggy outside. He felt light-headed and sick to his stomach. He denies losing consciousness and did not fall to the ground. He denies any chest pain. When you examine him, his eyes are sunken and he is tachycardic. What would be your next step in his management?
A. Electrocardiogram (ECG) B. Measure his blood glucose C. Echocardiogram D. Give fluids and recheck his vital signs E. Stress test
> D has been selected by the expert.
A. This choice is incorrect. While an EKG would be useful to assess for possible arrhythmia (long QT syndrome, WPW) or hypertrophic cardiomyopathy, the patient did not have a syncopal event or chest pain. As a result, an EKG would not be the most appropriate next step in diagnosis.
B. This choice is incorrect. While the patient has had poor PO intake, hypoglycemia is relatively uncommon in an otherwise healthy young male. In addition, we would expect symptoms including diaphoresis, anxiety, and tremulousness, as opposed to dizziness.
C. This choice is incorrect. Obstructive outlet pathology is less likely given the presentation, as no syncopal episode occurred. An echocardiogram may be indicated, but it would not be the next step in the evaluation.
D. This choice is correct. The patient is likely dehydrated given the dizziness without loss of consciousness in the setting of poor PO intake, hot weather and exercise. As the symptoms occurred while he was upright, the likely mechanism is vasovagal. His sunken eyes and tachycardia are signs of moderate to severe dehydration. Since this is a clinical diagnosis, fluids should be given with subsequent rechecking of heart rate and blood pressure to confirm the diagnosis.
E. This choice is incorrect. A cardiac etiology like hypertrophic cardiomyopathy is less likely given the presentation, and a stress test would not be the next step in the evaluation.
Claire is a 16-year-old female who presents for birth control management. Her review of symptoms is unremarkable except for chest pain. When you ask her more questions, she reveals the pains are intermittent, on and off for the past couple months. It is not associated with exertion, sharp, and well localized at the left sternal border. It is very brief, lasting only a few seconds, during which she says she sometimes notices it gets worse when she breathes in. She denies recent URI or viral illness. The family history is negative for early cardiac disease. Her vital signs and physical exam are normal. Which is the next best step in management?
A. ECG B. Reassurance C. Referral to a cardiologist D. Fast ultrasound of pericardial window E. Chest x-ray
> B has been selected by the expert.
A. An ECG would be next if you suspected a cardiac abnormality. This would be more likely if the patient described something more like angina, a crushing chest pain or pressure, for longer periods of time rather than a few seconds, and aggravated by exercise.
B. Based on the history, and assuming your physical exam is unremarkable, this sounds most suggestive of precordial catch syndrome, the most common cause of chest pain in an adolescent. No further workup is needed.
C. Associated symptoms of syncope or palpitations would suggest a more severe cardiac abnormality and would prompt a referral to a cardiologist. The vignette states the rest of the ROS is negative and this would not warrant further evaluation now by a cardiologist.
D. This would be the management if you suspected pericarditis or a pericardial effusion, but she denies recent URI or viral infection, and your PE is not notable for a pericardial friction rub suggestive of pericarditis. Her vitals are also stable and you might expect fever in pericarditis.
E. A chest x-ray would be indicated if you suspected pneumonia that potentially might present with chest pain. However, the vignette didn’t mention anything about cough and the patient is afebrile, making this less likely.
Adam’s birth weight plots above the 90th percentile for his gestational age of 36 weeks. How would you classify Adam? Select the single best answer.
A. Appropriate for gestational age (AGA)
B. Large for gestational age (LGA)
C. Small for gestational age (SGA)
B (90th and above is considered LGA)
Adam is a 2-hour-old infant born at 32 weeks’ gestational age via spontaneous vaginal delivery to a healthy mother with negative group B streptococcus status. There was no premature rupture of membranes and no meconium in the amniotic fluid. His Apgars were 8 at one minute and 9 at five minutes. Over the last two hours he has become progressively tachypneic. On physical examination he is large for gestational age. His vital signs are respiratory rate 75, temperature 36.5 C and heart rate is 130 beats per minute. His lung exam is remarkable for intercostal and subcostal retractions, grunting, and equal breath sounds. His heart exam reveals normal rhythm, normal S1 and S2, no murmurs, and normal peripheral pulses and capillary refill. Which of the following is the most likely cause of the patient’s condition?
A. Transient tachypnea of the newborn (TTN) B. Pneumothorax C. Congestive heart failure D. Respiratory distress syndrome E. Sepsis
> D has been selected by the expert.
A. Transient tachypnea of the newborn (TTN) is much more common in infants born to diabetic mothers. TTN is unlikely because he is 32 weeks, very premature, and was born via NSVD. RDS is much more likely, although TTN is still a possibility and would need to evaluated with a CXR.
B. Pneumothoraces are uncommon, but should always be considered in an infant with respiratory distress. Good bilateral air entry argues against this diagnosis.
C. Congestive heart failure is an important cause of tachypnea. Adam has a normal cardiovascular exam, with no murmur, normal pulses, and good capillary refill. Infants with congestive heart failure usually present with the triad of tachypnea, tachycardia, and hepatomengaly.
D. Respiratory distress syndrome (RDS) causes tachypnea and is therefore an important consideration in this case. RDS is more common in premature infants. Given the lack of history of maternal diabetes, an NSVD birth, and few risk factors for sepsis other than prematurity, Adam is likely to have RDS.
E. Infants may present with respiratory distress from sepsis or from pneumonia. In this case, Baby Adam has a normal temperature, good blood pressure, and normal perfusion. While less likely, this diagnosis should always be considered in infants with respiratory distress.
A 3-hour-old infant boy, born by C-section at 36 weeks to a 30-year-old G1P1 with Apgars of 8 and 9 at 1 and 5 minutes, respectively, is found to be tachypneic in the newborn nursery. His mother has a history of Type II diabetes that was poorly controlled during her pregnancy. She was compliant with prenatal vitamins and took no other drugs during her pregnancy. Prenatal labs, including GBS, were negative. The mother’s membranes ruptured 9 hours prior to delivery, she was afebrile, and the amniotic fluid had no meconium. On physical exam, the infant is large for gestational age. He has good air movement through the lungs bilaterally, without retractions or nasal flaring. He appears well perfused with normal cardiac exam. He is not in a flexed posture and has a weak suck reflex. A screening test at 3 hours of life reveals blood glucose of 39 mg/dL. What is the most likely diagnosis?
A. Hypoglycemia B. Transposition of the great arteries C. Transient tachypnea of the newborn D. Neonatal sepsis E. Pneumothorax
> A has been selected by the expert.
A. Hypoglycemia is a common presentation in an infant born to a diabetic mother with poor glucose control during her pregnancy. The increase in maternal serum glucose stimulates fetal pancreatic beta cells to increase insulin production, and this hyperinsulinemic state leads to hypoglycemia when the placental glucose supply is discontinued after delivery. At < 4 hours of life, a glucometer reading of < 25 mg/dL without symptoms or < 40 mg/dL with symptoms would require intervention to correct the hypoglycemic state. This infant has signs of hypotonia, with absence of flexed posture and weak suck, and a blood glucose reading of 39 mg/dL, making hypoglycemia the most likely diagnosis.
B. Transposition of the great arteries is a congenital heart defect in which the aorta and pulmonary artery are switched, resulting in poorly oxygenated blood pumped into the systemic circulation. The infant with transposition is generally cyanotic and will be in respiratory distress. This congenital defect is usual accompanied by a VSD, and maternal diabetes is a risk factor. This infant appears well perfused on exam, and has no murmurs on cardiac exam, making transposition a less likely diagnosis.
C. Transient tachypnea of the newborn (TTN) is a condition characterized by delayed clearance of amniotic fluid from the infant’s lung following birth (persistent postnatal pulmonary edema) resulting in respiratory distress. Infants born by C-section and to diabetic mothers are at an increased risk of TTN. X-ray findings include “wet” appearing lungs with significant perihilar streaking, interstitial and alveolar fluid, and fluid in the pleural space and along the fissures. TTN generally resolves within 24 to 48 hours and is treated symptomatically.
D. Neonatal sepsis is most commonly caused by GBS, Listeria and E. coli, transmitted from mother to baby. Additional risk factors include premature rupture of membranes (> 18 hours prior to delivery), preterm delivery, and chorioamnionitis. Infants may present with fever, trouble breathing, jaundice, and lethargy. Our infant is premature and tachypneic, but he is afebrile with normal Apgars and no evidence of altered level of alertness. Furthermore, mother was GBS negative, afebrile (no chorioamnionitis), with no premature rupture of membranes, making this diagnosis less likely.
E. Pneumothorax is collapse of lung tissue secondary to air accumulation in the pleural space. Risk factors for pneumothorax in an infant include previous intubation or underlying lung disease (such as severe respiratory distress syndrome). Characteristic physical exam findings include asymmetric breath sounds or decrease in breath sounds on one side. This infant has good air movement in bilateral lung fields, making this diagnosis less likely.
A male infant weighing 3200 grams is born to a G1P1 female at 39 weeks’ gestational age via planned C-section. Maternal PMH is unremarkable, and GBS status is unknown. Apgars are 7 and 8 at 1 and 5 minutes of life, respectively. The delivery is uncomplicated, and the infant initially appeared in good condition. However, one hour following delivery the infant develops increasing respiratory distress. RR is assessed as 90 breaths/min. All other vital signs are within normal limits. On exam, the infant is acyanotic with rapid respirations and robust capillary refill. Chest x-ray shows bilateral lung fields with the appearance of “a radio-opaque line of fluid in the horizontal fissure of the right lung.” No air bronchograms are noted. What is the most likely etiology of the infant’s respiratory distress?
A. Transient tachypnea of the newborn (TTN)
B. Respiratory distress syndrome (RDS)
C. Neonatal sepsis
D. Meconium aspiration
> A has been selected by the expert.
A. Transient tachypnea of the newborn (TTN) is the most likely underlying etiology. This condition is caused by residual fluid in the infant’s lungs following delivery, and usually resolves within several days. It is more common in babies delivered via C-section, as the normal mechanical force of labor that helps expel fluid from the lungs is lacking. Babies with TTN and other forms of respiratory distress are often unable to nurse and require feeding via NG tube until respiratory status stabilizes.
B. Respiratory distress syndrome (RDS) is less likely than TTN in this case. RDS is more common in premature infants and infants born to diabetic mothers. On chest x-ray, RDS is characterized by a ground-glass appearance and air bronchograms.
C. Neonatal sepsis is possible, especially given the mother’s unknown GBS status, but relatively unlikely compared to the other options, especially given the mode of delivery. Sepsis can certainly cause respiratory distress and, if suspected, should be promptly evaluated with screening labs and blood cultures. Neonatal sepsis is also more common with prolonged rupture of membranes (PROM) > 18 hours prior to delivery.
D. Meconium aspiration can lead to respiratory distress, but seems less likely in this case given the infant’s delivery via C-section. Additionally, meconium aspiration is more common when meconium is found in the amniotic fluid and/or products of conception. No mention of this was made in the above case description.
Adam is a newborn male who was just born to a G2P1 mother at 36.2 weeks’ gestation via a vaginal delivery. The mother reports that she did not receive prenatal care because she did not have insurance. She says that she thinks her “water broke” about two days ago, but she did not have any contractions after that, so she decided not to come to the hospital. She did not start having contractions until 19 hours before she delivered. After delivery, Adam did not cry vigorously, was tachypneic, cyanotic, and febrile to 100.5 F. Amniotic fluid did not contain meconium. His chest x-ray is normal. Given Adam’s birth history, what is the most likely cause of his symptoms?
A. Transient tachypnea of the newborn (TTN)
B. Sepsis secondary to prolonged rupture of membranes
C. Meconium aspiration syndrome
D. Hypothermia
E. Pneumothorax
> B has been selected by the expert.
A. Transient tachypnea of the newborn (TTN) is a benign, self-limited condition caused by delayed clearance of lung fluid after birth. Patients with TTN usually have a classic chest x-ray that shows coarse fluffy densities that represent fluid-filled alveoli and/or fluid in the pleural space and a small amount of fluid in the fissures on the lateral view. Given Adam’s normal chest x-ray and fever, it is unlikely that Adam has TTN.
B. Prolonged rupture of membranes (PROM) is when the chorioamniotic membrane ruptures before the onset of labor. The main risks associated with PROM are preterm labor and delivery and neonatal sepsis. Adam’s mom said that her “water broke” two days ago, which indicates that she had PROM. Adam’s mother also did not receive prenatal care; therefore, she did not receive any of the prenatal screening tests that she should have, which increases the likelihood that she has an infection that could have potentially been transferred to Adam after the rupture of her membranes. Adam’s history of PROM along with his fever and respiratory distress make this answer choice the best choice.
C. Meconium aspiration syndrome occurs when the baby passes meconium in utero and aspirates the meconium either in utero or at the time of delivery. Since it was noted that the amniotic fluid did not contain meconium, it is less likely that Adam is suffering from meconium aspiration syndrome, even though he has symptoms of respiratory distress that can often be seen in meconium aspiration syndrome.
D. Hypothermia can cause tachypnea of the newborn, especially premature newborns. However, hypothermia is less likely in Adam given his fever of 100.5 F.
E. Pneumothorax is less likely in this case because of Adam’s normal chest x-ray, but is important to consider in a tachypneic newborn. The presence of Adam’s fever also makes pneumothorax less likely to be the sole cause of his symptoms.
A newborn baby boy is born at 30 5/7 weeks’ gestation after induction of labor for the severe maternal preeclampsia. He is noted to have subcostal and intercostal retractions, grunting, nasal flaring, persistent cyanosis, and tachypnea 30 minutes after delivery. Apgars were 6 (-2 for color, -1 for breathing and -1 for tone) and 7 (-2 for color and -1 for breathing) at 1 and 5 minutes, respectively. Due to lack of prenatal care and the mother’s presentation with severe preeclampsia, betamethasone x 1 was given during induction, but she did not receive a second dose prior to delivery. A chest x-ray is obtained, which reveals diffuse ground-glass appearance and air bronchograms bilaterally. What is the most likely diagnosis?
A. Meconium aspiration syndrome (MAS)
B. Respiratory distress syndrome (RDS)
C. Persistent pulmonary hypertension (PPHN)
D. Transient tachypnea of the newborn (TTN)
E. Bronchopulmonary dysplasia (BPD)
> B has been selected by the expert.
A. This choice is incorrect. Although the presence or absence of meconium was not noted in the case, it is known that from 20 to 34 weeks’ gestation, the fetus will pass meconium infrequently. Most cases of MAS are in term or post-term infants. On chest x-ray, we might see overdistention of the lung or other sequelae, such as pneumothorax.
B. This choice is correct. The baby boy is preterm, and his mother received only one dose of betamethasone, which puts him at increased risk for developing infant RDS, which is caused by insufficient surfactant. His physical exam and chest x-ray findings are consistent with RDS.
C. This choice is incorrect. PPHN generally occurs in babies born after 34 weeks. There are several causes for PPHN: underdevelopment, maldevelopment, and maladaptation. Underdevelopment of the lungs can be secondary to congenital diaphragmatic hernia, oligohydramnios in utero, IUGR, or renal agenesis. The underdevelopment causes increased pulmonary vascular resistance and has a poor prognosis. Maldevelopment involves remodeling of pulmonary vasculature and is associated with post-term delivery and meconium aspiration syndrome. Maladaptation can be caused by infection with GBS. Vasoactive mediators are activated by bacterial phospholipids, causing an increase in pulmonary vascular resistance.
D. This choice is incorrect. TTN is a disorder of delayed reabsorption of fluid in the newborn’s lungs. Prematurity, delivery by C-section, being large or small for gestational age, or having a diabetic mother are all risks. In order to be diagnosed with TTN, the baby would need to show improvement within several hours. Although this is on the differential for the newborn baby’s condition based on clinical presentation, a chest x-ray should have shown perihilar streaking and other evidence of interstitial fluid.
E. This choice is incorrect, because BPD is the result of prolonged mechanical ventilation. Our patient is at risk for developing this syndrome if he requires intubation. Chest x-ray may show atelectasis, inflammation, or pulmonary edema. With severe disease, the chest x-ray may reveal fibrosis and hyperinflation.
Which of the following causes of hemolysis (and possible jaundice) may be hereditary?
A. G6PD deficiency B. Spherocytosis C. Elliptocytosis D. Alpha thalassemia E. Beta thalassemia F. Sickle cell anemia G. Pyruvate kinase deficiency
All of them!
Because many causes of hemolysis are hereditary, a family history of anemia or jaundice can provide important information.
Hemolysis leading to elevated circulating bilirubin and possible jaundice can be caused by a variety of disorders in the red blood cell, including:
Intrinsic cell membrane defects (such as spherocytosis and elliptocytosis)
Enzyme disorders (such as G6PD deficiency and pyruvate kinase deficiency)
Hemoglobinopathies (such as the thalassemias and sickle cell anemia)
These disorders have varied modes of inheritance (X-linked, autosomal dominant, autosomal recessive) and may be found with greater prevalence among certain ethnicities, or in certain parts of the world. For example, glucose-6-phosphate dehydrogenase (G6PD) deficiency, an X-linked recessive disorder, is more common in families of Mediterranean or West African origin than in other ethnic groups.
Hemoglobinopathies are not commonly associated with neonatal jaundice.
Which of the following options would you recommend if breast engorgement becomes a problem for Meghan’s mother? Select all that apply.
A. Use warm compresses before breastfeeding and cold compresses between feedings to relieve the discomfort.
B. Use manual or mechanical expression of the areola to relieve fullness and facilitate latch-on.
C. Have the baby nurse frequently to relieve breast engorgement.
D. Use Tylenol #3 (acetaminophen and codeine) as needed for the pain.
A, B, C.
What supplements are recommended for exclusively breastfed infants younger than 6 months of age? Select all that apply.
A. Calcium B. Fluoride C. Magnesium D. Folic acid E. Vitamin A F. Vitamin D G. Vitamin E
F only.
Vitamin D
To avoid the development of rickets, exclusively breastfed infants need vitamin D supplementation in the first 6 months of life.
Supplementation with 400 IU of vitamin D should be initiated within days of birth for all breastfed infants.
Infants who are not breastfed should also receive supplementation with 400 IU of vitamin D if they do not ingest at least 1 L of vitamin D-fortified formula daily. (The Canadian Paediatric Society recommends 800 IU intake per day-from all sources-between October and April depending upon where the family lives.)
Remember, sunlight is required to hydroxylate vitamin D.
Vitamin D deficiency rickets generally appears between 6 and 24 months and responds to treatment with vitamin D.
Iron
Although a 2010 AAP clinical report recommended iron supplementation for all exclusively breastfed infants beginning at 4 months of age, this recommendation has been debated. Most pediatricians do recommend the addition of iron-containing foods to the infant’s diet, starting at age 6 months.
Most standard formulas are iron-fortified.
Fluoride
Breast- and bottle-fed infants both should receive fluoride supplements after the age of 6 months if the water supply lacks fluoride (< 0.3 ppm).
Note that most bottled and filtered water has low fluoride levels.
What physical finding that could contribute to hyperbilirubinemia might you see on Meghan’s head if she had suffered birth trauma?
A. Cephalohematoma
B. Caput succedaneum
C. Bruising
A, C
A cephalohematoma (see photo) is a subperiosteal hemorrhage that is localized to the cranial bone that was traumatized during delivery.
The swelling does not extend across a suture line.
As the blood is reabsorbed from the cephalohematoma it will contribute to hyperbilirubinemia.
Bruising on the head or elsewhere on the body from birth trauma or any other bleeding can also lead to increased bilirubin production because blood extravasated into tissues will be broken down and converted to bilirubin.
A caput succedaneum is an edematous swelling over the presenting portion of the scalp of an infant and is commonly seen in babies born vaginally in vertex position. Because the scalp overlies the periosteum, this boggy swelling crosses suture lines, easily differentiating it from a cephalohematoma.
A caput succedaneum (B) is an edematous swelling over the presenting portion of the scalp of an infant.
It overlies the periosteum and the swelling crosses suture lines.
The swelling consists of serum and would not cause hyperbilirubinemia.
Dr. Harris explains that breastmilk jaundice is the most likely diagnosis. He challenges you to consider what labs, if any, are absolutely essential to draw at this point:
Of the following, which seem critical?
A. Total serum bilirubin B. Direct bilirubin C. TSH D. CBC E. Direct Coombs
> A has been selected by the expert.
Total serum bilirubin (A) is essential to judge the severity of Meghan’s hyperbilirubinemia.
Most pediatricians would order a direct bilirubin level at least once during the evaluation of a persistently jaundiced infant, but not necessarily during the initial evaluation.
The direct bilirubin would be especially indicated if the baby were ill-appearing (due to concern for infection as an etiology), or had acholic stools and/or jaundice that persisted beyond three weeks (due to concern for possible biliary atresia or other forms of cholestatic jaundice).
TSH would be helpful if congenital hypothyroidism were a significant concern based on history or exam. An additional lab draw is not necessary, however. Meghan’s newborn screen results will help rule out hypothyroidism.
Although a normal CBC and Direct Coombs would provide reassurance that sepsis and hemolysis are not present, you have already excluded these based on the timing of her jaundice, history, and exam.
Based on Meghan’s total serum bilirubin result and the likely diagnosis of breastmilk jaundice, which one of the following management plans is most reasonable?
A. Admit for phototherapy.
B. Continue breastfeeding and recheck total bilirubin level in 24 hours.
C. Interrupt breastfeeding and feed formula for as long as jaundice persists.
D. Supplement 1-2 ounces of formula after every breastfeeding for as long as jaundice persists.
> B has been selected by the expert.
The most reasonable approach is to continue breastfeeding and recheck total bilirubin in ~24 hours (B).
Meghan’s total bilirubin of 17.0 mg/dL (291 μmol/L) is in the high-intermediate risk zone on the Bhutani nomogram.
Because she is a full term, clinically well newborn with no risk factors for neurotoxicity, phototherapy (A) is not indicated at this point.
The AAP recommends that, if possible, breastfeeding should be continued in jaundiced infants. Interrupting breastfeeding and feeding formula as long as baby is jaundiced (C) should not be advised.
If breastmilk jaundice persists, stopping breastfeeding for 24 hours (while having mom pump and store milk) may help bring bilirubin levels down and avoid the need for ongoing monitoring of total serum bilirubin.
Supplementation with formula (D) may be indicated in cases of breastfeeding (not breastmilk) jaundice.
A 4-day-old baby boy presents for his first pediatric well child visit. His birth history consists of an uncomplicated normal spontaneous vaginal delivery after 7 hours of labor-no vacuum or forceps assistance were used. The patient is the first child to a 30-year-old mother of Mediterranean descent. Mom is very concerned that her baby has started to look “yellow” since leaving the hospital. She has been breastfeeding every 2-3 hours and says that the baby latches on for 1-5 minutes for each feed. He has had few wet diapers, and mom is concerned he is not getting enough to eat. Which of the following would most aid in narrowing the differential diagnoses?
A. Newborn screen results B. Fractionated bilirubin C. WBC D. Blood smear E. No further workup is needed, as this is likely physiologic jaundice
> B has been selected by the expert.
A. The newborn screen result normally takes a few weeks to process. However, even if the results were available, they might suggest two possibilities in the differential diagnosis of jaundice: hypothyroidism or G6PD deficiency. Even though these diagnoses are important to rule out, the test would not provide more information about the infant’s current state of health or the next step in treatment.
B. The test that will give you the most information at this juncture is a fractionated bilirubin. With the knowledge of the total serum bilirubin (TSB) and direct serum bilirubin, one will be able not only to narrow the differential (hemolysis vs. obstruction), but also to guide treatment (i.e., indirect serum bilirubin may be above phototherapy level). TSB can also indicate if the situation requires more drastic measures, such as a transfusion exchange.
C. A WBC could be helpful if the infant is at risk or showing signs of infection or sepsis. The child did not have a complicated birth history, and now has no signs of fever or distress. Although it may be helpful to check a CBC and know the hemoglobin level, in case of hemolysis, a WBC will not help narrow the differential as much as a fractionated bilirubin given the above presentation.
D. A blood smear would be helpful to diagnose hemolysis or RBC membranopathies. However, a blood smear would not be the most useful in narrowing the wide range of possibilities in the current differential, because it would help with identifying hemolytic causes. At this juncture, a fractionated bilirubin would be the most useful test to order first.
E. Although physiologic jaundice is the most likely cause of jaundice at this age for an infant, there is not yet enough information to rule out a more serious cause. Because the mother is of Mediterranean descent, it is very possible that the cause of jaundice is G6PD deficiency, so further workup is definitely necessary. At the very least, a fractionated bilirubin is needed, and then likely a CBC.
A concerned mother brings her 7-day-old son to your office after noticing yellowing of his skin for 2 days. She has also noticed he has not been gaining weight since she brought him home from the hospital 5 days ago. This is her first son and she has been trying to do everything perfectly, including breastfeeding him, since she was told that breast milk provides adequate nutrients and other healthy benefits, like antibodies and growth factors. However, upon further questioning, she is feeding him only 6 times a day for 10 minutes each time. She admits her breasts often feel full and are not relieved by nursing. He was born full term by spontaneous vaginal delivery but had a hard time sucking with breastfeeding. Upon exam, he looks dehydrated and appears to have jaundice of the face and chest. He has also lost > 10% of his birth weight. What could be the cause of his jaundice?
A. Breast-milk jaundice B. Physiologic jaundice C. Sepsis D. Breastfeeding jaundice E. Crigler-Najjar syndrome
> D has been selected by the expert.
A. Breast-milk jaundice is incorrect, because although it is a cause of jaundice associated with breastfeeding, it is NOT a result of low milk volume. In the above vignette, the infant does not appear to be breastfeeding well, which is multifactorial-poor suck and low number of feeds for the infant. Breast-milk jaundice would appear only if the infant were feeding well, although it is not completely understood what causes this form of jaundice.
B. Physiologic jaundice is incorrect because this jaundice usually peaks at 3-4 days of life to full-term, healthy infants. This type of jaundice is not associated with breastfeeding but could be from numerous factors such as increased bilirubin production from short-lived RBCs or lack of intestinal flora to metabolize bile. No treatment is required for this type and it typically resolves on its own.
C. Sepsis is incorrect because the infant does not clinically appear sick or toxic, and jaundice would be just one sign of a serious infection. Other symptoms may include respiratory distress, lethargy, poor feeding, vomiting, apnea, and temperature instability. Sepsis causes an elevated total and direct bilirubin and is highly unlikely when jaundice is the only symptom. Breastfeeding can have some protection against infection.
D. Breastfeeding jaundice is the correct answer because it usually appears early in the first week of life and is caused by various factors, including poor breast milk intake. A decreased milk supply leads to limited enteral intake and can lead to increased enterohepatic circulation. Increased enterohepatic circulation describes the process where unconjugated bilirubin is reabsorbed in to the bloodstream where it binds to albumin and is recirculated.
E. Crigler-Najjar syndrome is an incorrect choice because it appears within the first days of life and persists thereafter and is a relatively rare disease. This type of jaundice is caused by decreased bilirubin clearance from deficient or completely absent UDPGT (the enzyme that conjugates bilirubin with glucuronide to make it water-soluble and able to undergo excretion into bile).
A 5-day-old infant presents with a chief complaint of jaundice. As you obtain a careful history and physical examination, which of the following would NOT be a risk factor for jaundice in this infant?
A. Mediterranean origin
B. Prolonged labor with use of forceps during the delivery
C. Mother is type O+ and baby is type B
D. Phenylketonuria
E. Poor breastfeeding during first few days of life
> D has been selected by the expert.
A. Families of Mediterranean origin have higher risks of hereditary diseases that can cause jaundice. Glucose-6-phosphate dehydrogenase (G6PD) deficiency and the thalassemias are more common in Mediterranean families. Both cause hemolysis of RBCs, which results in an increased of jaundice.
B. Difficult deliveries and birth trauma may result cephalohematomas, or hemorrhage of blood between the skull and the periosteum. These result from the rupture of blood vessels crossing the periosteum, usually caused by a prolonged second stage of labor or the use of forceps or other instrumentation during delivery. As the cephalohematoma resolves over weeks and the blood is reabsorbed, the breakdown of RBCs from the hematoma can result in increased bilirubin levels.
C. ABO compatibility is a common cause of hemolysis in newborns. Since the mother has blood type O, she makes IgG antibodies to A-antigens and to B-antigens, leading to hemolysis of the fetal red blood cells. This can be detected by a positive direct Coombs test.
D. Phenylketonuria (PKU) is an autosomal recessive metabolic disorder due to a mutation in phenylalanine hydroxylase, which is required to convert phenylalanine to tyrosine. PKU leads to buildup of phenylalanine in the brain, leading to mental retardation, seizures, and death if not detected and treated early. It is not associated with jaundice.
E. Poor breastfeeding during the first few days of life and decreased enteral intake may result in breastfeeding jaundice. This may be caused by the mother who does not make enough milk, or by the infant with inadequate feeding or intake. Decreased intake leads to decreased motility of the GI tract and retention of meconium. The meconium contains β-glucuronidase, which hydrolyzes the conjugated bilirubin to an unconjugated form, which is reabsorbed and re-circulated into the blood through the enterohepatic circulation, increasing bilirubin levels in the blood.
A 3-week-old baby boy is brought to his pediatrician with a chief complaint of light tan-colored stools and worsening jaundice. His is exclusively breastfed and has 6-8 wet diapers per day. On exam, he appears to have scleral icterus and jaundice. Upon further workup, he is found to have an elevated direct bilirubin. What is his most likely diagnosis?
A. Biliary atresia B. Breastfeeding jaundice C. G6PD deficiency D. Physiologic jaundice E. Caput succedaneum
> A has been selected by the expert.
A. Biliary atresia can present anytime between birth and 8 weeks of age, but usually occurs after 2 weeks of age. Jaundice is usually the first presenting finding, along with acholic stools, dark urine (from increased bilirubin excretion) and hepatosplenomegaly if the problem goes unrecognized. Laboratory values classically show an increased level of direct or conjugated bilirubin > 2 mg/dL. If biliary atresia is confirmed with further laboratory testing and imaging, surgical intervention must be pursued as soon as possible.
B. Breastfeeding jaundice normally occurs within the first week of life, most often because of decreased intake leading to dehydration and increased enterohepatic circulation. This patient is exclusively breastfed, but his jaundice began at approximately day 16 of life. Also, breastfeeding jaundice increases unconjugated bilirubin levels, making this answer choice less likely.
C. G6PD is an X-linked inherited disorder. The severity of this disorder is dependent upon the degree of deficiency of the enzyme, but may present with neonatal unconjugated hyperbilirubinemia. However, laboratory findings show hemolytic anemia in the symptomatic state. Hemolysis is usually elicited by drugs, most notably primaquine and dapsone, as well as fava beans.
D. Physiologic jaundice peaks at 3-4 days of life and generally resolves within a day or two. This patient is well beyond that age, making this answer less likely.
E. Caput succedaneum is caused by an increase in serum above the periosteum of an infant that crosses suture lines (as opposed to cephalohematoma which does not cross suture lines). The increased serum bilirubin does not usually lead to significant hyperbilirubinemia.
A two-month-old female presents to clinic for a well-baby checkup. Mom has been happy because the “baby rarely cries and sleeps all the time.” On exam, the baby has yellowing of the skin, decreased activity, appears to have decreased tone, and a large anterior fontanel. What is the most likely diagnosis?
A. Sepsis B. Congenital adrenal hyperplasia C. Congenital hypothyroidism D. Shaken baby syndrome E. Neonatal lupus
C
A. Sepsis is a blood infection of the infant. Signs and symptoms may include body temperature change, changes in respiration, increased or decreased heart rate, reduced movement, reduced feeding, low blood sugar, seizures, and jaundice.
B. Infants with congenital adrenal hyperplasia often have abnormal genitalia (females), poor feeding, vomiting, dehydration, and electrolyte changes.
C. Congenital hypothyroidism may not be clinically evident until 6 weeks of age due to circulating maternal thyroid hormone transmitted from the placenta. Signs and symptoms of congenital hypothyroidism include feeding problems, large fontanels, hypotonia, large tongue, coarse cry, and frequently an umbilical hernia. Congenital hypothyroidism should be picked up on routine neonatal screening.
D. Shaken baby syndrome may result in significant head trauma, including subdural hemorrhage, hypoxic/ischemic brain injury, and retinal hemorrhage.
E. Neonatal lupus should be considered when the infant’s mother has lupus. Cutaneous findings may be present at birth or may develop within the first 2 to 5 months of life. These findings include erythematous plaques, telangiectasias, or atrophic lesions. Affected infants may have cardiac abnormalities or conduction deficits. Hematologic disturbances may occur within the first 2 weeks of life.
Jade is a 2-week-old female who was born at home and received no newborn screenings for congenital disease. Her mother brought her to the pediatrician’s office concerned that her daughter appeared to be jaundiced and was constipated, tired, and not feeding well most of the time. Physical exam was notable for enlarged fontanels, jaundice without bruising, hypotonia without tremor or clonus, and an umbilical hernia. There was no sign of virilization, no abnormal facies, and no history of vomiting. Review of systems was otherwise negative except as stated above. Which of the following is the most important next step in Jade’s management?
A. Glucose and electrolyte supplementation
B. Glucocorticoid and mineralocorticoid supplementation
C. No treatment needed
D. Consult with pediatric endocrinologist and start treatment with 10 to 15 mcg/kg/day of crushed levothyroxine in liquid, and follow up every 12 months
E. Empiric antibiotics after collection of blood, urine, and CSF cultures.
D
A. This choice is incorrect because this is the treatment of choice for acutely ill children with dehydration, hypoglycemia, and perhaps infants with congenital adrenal hyperplasia, not congenital hypothyroidism as is most likely in this infant.
B. This is the recommended treatment for corticoid and mineralocorticoid deficiency as seen in congenital adrenal hyperplasia. Clinical evaluation of this patient does not show virilization of this female infant, commonly seen in congenital adrenal hyperplasia. However, it would still be important for our patient in this case to undergo newborn screening, which includes screening for this disorder.
C. This choice is incorrect because congenital hypothyroidism requires treatment within 2 weeks of onset of symptoms to mitigate severe brain damage and cognitive impairment. No treatment is needed for children born with hypothyroxemia of prematurity without TSH elevation.
D. This choice is correct because the American Academy of Pediatrics recommends this treatment regimen for infants age 0 to 6 months old. Dosing is based upon age and weight. It would also be important to consult with a pediatric endocrinologist to evaluate the short and long-term treatment plan. In addition, the specialist could also recommend screening for other autoimmune disorders.
E. This choice is incorrect. Although sepsis must always be considered in a neonate with jaundice, there is no indication of bacterial infection in this infant with other signs and symptoms of congenital hypothyroidism.
A 6-week-old infant girl whose family recently immigrated from Mexico is brought to clinic for “excessive sleepiness.” The mother states the infant is not easily aroused for feedings and is not as active as she was previously. She is also concerned about her daughter’s large “outtie” belly button. On exam, the patient is afebrile and jaundiced, with a puffy myxedematous face. The fontanels are large but flat. There is a large umbilical hernia. When asked about the results of a newborn screening exam, mom states that the screening was never performed. What would be an expected abnormal lab value(s) associated with her condition?
A. Low sodium, high potassium
B. Glucose < 40 mg/dL
C. High WBC with bandemia
D. High TSH, low T4
D.
A. A low sodium and high potassium in a lethargic infant would suggest the diagnosis of congenital adrenal hyperplasia (CAH), characterized by a decreased production of cortisol and aldosterone. Low aldosterone results in decreased stimulation of the H/K exchange in the collecting duct, hence loss of sodium, retention of potassium, and dehydration. In combination with low cortisol levels, patients in adrenal crisis may progress to shock, and death is not treated. The usual age of presentation is 1 to 2 weeks of age. Initial laboratory studies would include serum electrolytes, renin, cortisol and cortisol precursors, androgens, and glucose levels. Note that low cortisol will also impact gluconeogenesis and glycogenolysis. This diagnosis would be less likely in a non-viralized female.
B. Symptoms of hypoglycemia in the neonate may include jitteriness, tremors, hypotonia, poor feeding and seizures. Management includes STAT glucose levels and intervention with parenteral or oral glucose, as indicated. The work up will include laboratory studies to rule out hyperinsulinism (IDM, insulinoma, prematurity), increased metabolic demand (polycythemia, sepsis, asphyxia), and inborn errors of metabolism (galactosemia, glycogen storage diseases, maple syrup urine disease). This patient did not present with jitteriness, tremors, or seizures.
C. Sepsis should always be considered in lethargic neonates. While septic infants may present with fever or hypothermia, they may also be afebrile. This child did not appear to be acutely ill or toxic in appearance.
D. Congenital hypothyroidism may present with poor feeding, constipation, jaundice (longer and more persistent than physiologic jaundice of newborn), mottled skin, large fontanels, hypotonia, hypothermia and an umbilical hernia. Later findings include a hoarse cry, macroglossia, and myxedematous facies. Patients usually remain asymptomatic until after 6 weeks of age, as maternal thyroid hormones may still be in younger infants. Patients with primary hypothyroidism will have high TSH and low T4 levels. The most common cause of primary hypothyroidism will be aplasia or hypoplasia of the thyroid gland, and-much less commonly-inborn errors of metabolism. Secondary or tertiary hypothyroidism (HPA dysfunction) will have both low TSH and low T4, and are relatively rare causes of hypothyroidism in infants.
A 45-day-old infant is brought in by his mother due to lethargy, constipation, and yellow skin color noted since birth. The mother and the baby moved to the U.S. from a foreign country that does not screen its newborns. The baby has been fed only formula since birth. Physical exam of the neonate reveals additional findings of large fontanelles, umbilical hernia, a large tongue, and abdominal distension. What is the next best step in diagnosis?
A. Phototherapy B. Exchange transfusion C. TSH D. Head ultrasound E. Obtain a family history of jaundice to rule in or out a defect in bilirubin metabolism
C
A. Phototherapy is the treatment for physiologic jaundice, which peaks at three to four days and resolves by the fourth or fifth day of life. Lethargy, macroglossia, and umbilical hernia are not known to be caused by or associated with physiologic jaundice. This constellation of physical exam findings is more consistent with untreated congenital hypothyroidism.
B. This choice is incorrect because this is the management of infants with hyperbilirubinemia approaching levels of concern for kernicterus.
C. This choice is correct because the constellation of baby’s problems is best accounted for by untreated congenital hypothyroidism. Unfortunately, severe mental retardation is unavoidable at this point because this condition should have been treated since birth. In the U.S., it would have been detected on the newborn screen.
D. This choice is incorrect as there is no indication of hydrocephalus or concern for bleeding.
E. This choice is incorrect because-although defects in biliary metabolism such as Gilbert’s syndrome, seen in 5% of the population, can cause harmless jaundice-this patient has many other findings in addition to jaundice. Of the answer choices given, only congenital hypothyroidism fully accounts for the entire constellation of findings.
The parents of 5-month-old Tiffany are concerned about Tiffany’s decreasing oral intake over the past 4 days. They report that she has been sleeping more but seems to tire out when feeding; in fact, mom’s breasts have become quite engorged and she needs to pump to relieve the pressure. In addition to the sleepiness and poor feeding they report that she has not had a bowel movement in 3 days. She has no fever or respiratory symptoms. You note a weak cry on your exam, and a floppy baby when you try to sit her up. What additional finding are you likely to find on your exam?
A. Vesicular rash on her scalp B. Large tongue and umbilical hernia C. III/VI systolic murmur D. Absent deep tendon reflexes E. Cataracts and hepatosplenomegaly
D.
A. A vesicular rash may be seen in neonatal HSV infection, which can be a cause of encephalitis. It is less likely in this older age group and would most likely present with fever and possibly seizure.
B. Although congenital hypothyroidism can present with lethargy, constipation, and poor feeding, the infant would be less likely to present with these symptoms as late as 5 months of age.
C. Congenital heart disease may present with poor feeding, but a large VSD would likely present earlier and would not be associated with constipation and hypotonia.
D. This infant likely has infant botulism which usually presents in the first year of life with hypotonia, lethargy, constipation, weak cry and can eventually lead to respiratory failure. These infants will have absent DTRs.
E. An inborn error of metabolism can present with lethargy and poor feeding-and hepatosplenomegaly and eye findings may also be present-but this infant presented more acutely and at an older age than would be expected for a metabolic disorder.
A 6-month-old infant arrives in the ED with a 12-hour history of poor feeding, emesis, and irritability. On exam, she is ill-appearing with T 39.2 C, P 160 bpm, R 40 bpm, BP 80/50 mmHg. CBC shows WBC 11.2, Hgb 13.5, Plt 250. Urinalysis shows > 100 WBC per hpf, positive leukocyte esterase, and positive nitrites. She has no history of prior urinary tract infection. Chest x-ray is negative. Urine and blood cultures are pending. After bringing her fever down, she was still uninterested in drinking, but her exam improved, and you were confident she did not have meningitis, so an LP was not performed. Which of the following is the best next step in management?
A. Oral ampicillin B. Oral ampicillin + gentamicin C. Intravenous ciprofloxacin D. Intravenous ceftriaxone E. Intravenous piperacillin + tazobactam
> D has been selected by the expert.
A. This patient is too sick for oral treatment, so oral ampicillin would be insufficient. Also, there is rising resistance of E. Coli to ampicillin, so secondary coverage with gentamicin or some other aminoglycoside would be needed unless cultures proved the organism was sensitive to ampicillin alone.
B. Although parenteral and oral treatment produce similar outcomes in high quality RCTs, this patient is ill and refuses to drink and so requires parenteral antibiotics. IV ampicillin and gentamicin could be an appropriate choice for empiric coverage.
C. Ciprofloxacin could be used for complicated UTIs, but it has the potential for adverse reactions in young children so is reserved for patients > 1 year with complications such as resistant organisms or urinary tract anomalies
D. This patient’s presentation is suggestive of a UTI. Given the ill appearance, vital signs, and white count, Upper tract disease (pyelonephritis) should be strongly considered. A parenteral (IV/IM) third-generation cephalosporin is the best choice of those listed for pyelonephritis, given its excellent gram negative coverage (except for Pseudomonas).
E. IV piperacillin + tazobactam has excellent gram negative coverage with added Pseudomonas coverage, but it is highly expensive and Pseudomonas is unlikely to be the cause of a UTI in a child who is not regularly catheterized.
A 3-month-old male presents to the ED with a fever that started the previous day. Mother reports that he was fussy and had decreased oral intake. He had had five fewer diaper changes than usual. He had no vomiting, diarrhea, or respiratory difficulty. On physical exam his temperature is 101.6 F, pulse 110 bpm, RR 24 bpm, and BP 95/67 mmHg. The baby seems irritable and is not consolable by the parent. HEENT exam was significant for dry mucous membranes. Other than his irritability, the rest of the physical exam was unremarkable. CBC showed WBC 3.5, but was otherwise normal. BMP was within normal limits. Urinalysis showed positive leukocyte esterase, positive nitrite, and WBCs > 10/hpf. An LP was performed, and urine and CSF culture results are pending. The patient is placed on IV fluids and is started on cefotaxime. What is the next best step in evaluation?
A. Renal bladder ultrasound B. Kidney-ureter-bladder (KUB) x-ray C. Intravenous pyelogram D. VCUG E. Oral ampicillin
> A has been selected by the expert.
A. This infant has a fever without other respiratory symptoms. Meningitis and UTI must be considered in patients with fever. The only way to rule out meningitis is by lumbar puncture. This patient has a low WBC, suspicious for sepsis, and a UA that is highly suggestive of UTI. Empiric therapy should be started to cover common organisms including E.coli, P. mirabilis, and Klebsiella. Cefotaxime is reasonable empiric therapy. Renal ultrasound is recommended for all infants with pyelonephritis to assess for renal structural abnormalities or signs of obstructive uropathy (hydronephrosis).
B. KUB is not recommended for UTI.
C. Intravenous pyelogram would expose the patient to radiation and would not be recommended to screen for renal abnormalities.
D. VCUG screening is recommended only for recurrent UTI or when there is abnormal renal ultrasound.
E. The patient is already on parenteral antibiotics, so oral antibiotics would not be necessary. Also, ampicillin would not provide empiric coverage.
A 10-day-old boy is brought to the ED by his mother because of “fever.” Mom describes that the baby has been “sleepy” and feeding less vigorously than in the previous two days. She believes his urine output has also decreased. His birth history is notable for prolonged membrane rupture (about 32 hours), and maternal fever at the time of delivery. Prenatal and neonatal ultrasound revealed bilateral hydronephrosis. On exam, the infant is sleepy with a temperature of 38.5 C. A blood sample is sent for CBC, BMP, and culture. Attempts are made to obtain CSF and urine for analysis and culture, but only very small volumes of these fluids are obtained. Volume resuscitation is begun. Chest x-ray is performed with indeterminate results. What is the most appropriate next step?
A. Send samples for gram stains and begin parenteral empiric antibiotic treatment
B. Send the urine for urinalysis and the CSF for cell count, glucose and protein and begin parenteral antibiotic therapy
C. Admit for observation and continue supportive care
D. Send samples for culture and begin parenteral antiobiotic treatment
E. Attempt to obtain larger samples. Antibiotics should not be started until all needed results are pending.
> D has been selected by the expert.
A. Although sending samples for gram stain may give an indication of whether an infection is present, it will not give the same degree of information as would a culture with sensitivities.
B. Urinalysis and CSF profiles may help us make the diagnosis, but if positive in the absence of cultures, will commit us to a prolonged course of broad-spectrum and non-specific therapy.
C. Delay of therapy would not be indicated. In an infant younger than one month, fever with any suspicion of sepsis, whatever the source, requires immediate evaluation and initiation of antibiotic treatment.
D. Given the presentation of fever in a neonate who presents with sleepiness and poor feeding, samples should be sent for culture and the baby started on empiric antimicrobial therapy. This infant is likely to have a urinary tract infection, and urosepsis is certainly a possibility, especially given his known urinary tract anoamlies. We have no way of ruling out meningitis from this presentation, so antibiotics should be initiated at meningitic dosing. In an infant younger than one month, fever with any suspicion of sepsis, whatever the source, requires immediate evaluation and initiation of antibiotic treatment. Because infants at this age have immature immune systems, they do not localize infections as well as older children. An infection of the urinary tract may lead to bacteremia, which in turn may lead to CNS infection. Only cultures will give us the information required to determine the appropriate type length of antimicrobial therapy.
E. Given the consequences of significant bacterial infection in an infant this age, delaying therapy to obtain additional laboratory specimens is not appropriate.
A 6-month-old female is brought into the pediatrician’s office for three days of high fever, fussiness, and decreased appetite. The patient has not had any upper respiratory tract symptoms, vomiting, diarrhea, or rash. On physical exam the patient is fussy, has a RR of 28 bpm and a pulse of 160 bpm. She is febrile to 102.8 F (rectal). The patient is alert and fully moving all extremities. Apart from her vital signs, no other significant exam findings are noted. A CBC demonstrates leukocytosis of 17.0 cells x 103 / µL with elevated bands. What diagnosis is most likely?
A. Measles B. Bacterial meningitis C. Acute otitis media D. Urinary tract infection E. Roseola
> D has been selected by the expert.
A. Measles typically begins with a “prodrome” period featuring the “3 Cs” (cough, coryza, conjunctivitis)-none of which this patient has-along with high fever, often > 104 F, and general malaise and anorexia. On the 2nd to 4th day a maculopapular erythematous rash appears starting on the face/upper neck and spreading downward. Although infants receive their first vaccination against measles (the MMR) at 1 year of age, infants are generally protected unless they are exposed to older, unimmunized children who have the disease.
B. This answer is incorrect in this situation for several reasons. First, this patient is not toxic appearing, nor is her physical exam positive for any findings suggestive of meningitis (such as bulging fontanel or extreme irritability). Unlike older children or adults, classic meningeal signs will often not be present or will be difficult to appreciate on an infant. The patient’s CBC is significant for leukocytosis with elevated bands, suggesting a bacterial infection. On exam the patient is alert and responding well to her environment and does not demonstrate lethargy, respiratory distress or signs of ICP. A definitive lumbar puncture may be ordered if there is more suspicion for meningitis or if the patient’s status deteriorates in any way, and caretakers should be given a follow-up appointment as well as clear indications of when to seek care.
C. This answer is incorrect because while fever and fussiness can be possible signs of AOM in infants, there is no evidence of infection on physical exam such as inflamed, erythematous tympanic membranes, with bulging of the membrane indicating an effusion.
D. UTI, the most common bacterial illness in a female infant, is consistent with her high fever, fussiness, and decreased appetite. Her CBC suggests that she has a bacterial infection (leukocytosis and elevated bands). A sample of her urine should be obtained by catheterization and sent for urinalysis and culture.
E. Roseola often presents with a high fever, but also often with a viral prodrome. It is a diagnosis of exclusion at this point and should not preclude obtaining a urine sample in this child.
A 6-month-old female with normal birth and developmental history presents with fever for the past two days, fussiness, and decreased appetite. ROS is negative. No abnormalities are noted on the physical examination. A urinalysis from a bag specimen is positive for leukocytes and nitrite, which suggests the presence of a UTI; a culture from this sample is pending. The patient is ill-appearing, dehydrated, and unable to retain oral intake. She is hospitalized, receives a 20 cc/kg NS bolus and is placed on maintenance IV fluids with clinical improvement. What is the best next step for management of this patient?
A. Urinary catheterization
B. Renal bladder ultrasound
C. Begin parenteral antimicrobials
D. Midstream clean catch urine collection
E. Increase intravenous fluid administration rate to flush the kidneys
A
A. Urinary catheterization is correct . It is the best method for obtaining a specimen for culture that has not been contaminated by perineal bacteria, and for this ill child, you must determine the cause of the fever with accuracy.
B. Renal ultrasound may be indicated if the infant is found to have a urinary tract infection but is not indicated as part of the initial work up.
C. With a bag culture pending, you may have a contaminated/confusing culture result and may have to rely solely on clinical criteria to treat this presumed UTI.
D. Midstream clean catch specimen is incorrect because it is not ideal in a patient who has not been toilet-trained and cannot void on demand.
E. While it is important to provide hydration, this patient has responded well to the initial fluid administration, and there is no evidence that increasing this beyond normal recommendations is helpful.
A 5-year-old female, previously healthy, presents with an erythematous, vesicular rash on the palms and soles and a high fever for several days. Upon examination, she is also found to have ulcers in her mouth. A few days later, the fever and rash resolve. What is the most likely pathogen?
A. Herpes simplex virus 1 (HSV-1) B. HIV C. Enterovirus D. Human herpesvirus 6 (HHV-6) E. Group A strep
> C has been selected by the expert.
A. HSV-1 causes gingivostomatitis and can sometimes be accompanied by fever and malaise, but lesions on the hands and feet would be uncommon.
B. HIV infection can increase the risk of oral lesions secondary to infections by HSV-1 or Candida albicans, but again would be unlikely to cause lesions on the hands and feet. Furthermore, at this patient’s age (5 years), she is unlikely to be HIV-positive unless it was vertically transmitted from her mother.
C. This presentation is consistent with infection by cocksackie A, an enterovirus. Following an incubation period of three to five days, patients have fever, tender vesicles on their hands and feet, and oral ulcers. Sometimes the rash also occurs on the buttocks and the genitals. The infection resolves spontaneously within three days, and is spread from person to person via saliva, fluid from the vesicles, stool, or nasal discharge.
D. HHV-6 is the virus that causes roseola, which manifests as fever followed by a macular or maculopapular rash, but this rash begins on the trunk, eventually spreading to the extremities, and does not cause oral lesions.
E. Group A strep infection could cause fever and a rash with scarlet fever, but this rash is described as “sandpaperlike” with small papules, not vesicular, and is also not confined to the hands and feet.
A 2-year-old girl presents to the urgent care clinic with a 7-day history of high fever to 38.5 C, a maculopapular rash that began on the palms and soles of her feet, red eyes without discharge, and unilateral cervical adenopathy. What other symptom/sign might you discover on further history and exam?
A. Tonsillar exudates B. Headache C. Erythematous and edematous feet D. White spots on buccal mucosa E. Dysuria
> C has been selected by the expert.
A. Tonsillar exudates would be present in strep pharyngitis or tonsillitis. Given the prolonged fever, rash, lymph node involvement, and conjunctivitis, the disease process is more widespread than a simple tonsillitis. You should recognize this constellation of symptoms as Kawasaki disease.
B. Headache would be present in Rocky Mountain Spotted Fever. This seems reasonable, given the fever and rash that began on the palms and soles. However, the other findings suggest Kawasaki disease, so this is not the best answer.
C. The constellation of symptoms described suggests Kawasaki disease. The other two classic signs not mentioned are erythematous tongue (“strawberry tongue”), and erythema/edema of the extremities, which is the best answer here.
D. White spots on the buccal mucosa are also known as Koplik spots, which are pathognomonic for measles. The fever and conjunctivitis could be measles, but there is no cough or coryza. In addition, the rash typically starts at the head and moves downward, rather than starting on the hands and feet.
E. Although children with Kawasaki disease can have pyuria, it is not associated with dysuria, a symptom of a UTI, which would be highly unlikely given the other signs.
A 3-year-old male presents with fever to 103 F for the past week, injected eyes, and a refusal to walk for the past two days. On physical exam, you note conjunctival injection without pus or exudates bilaterally, prominent papillae of his tongue with redness as well as redness of his hands, and feet. He also has a new non-diffuse maculopapular rash on his torso that gets worse with fever. On examination of the swollen extremities, you are unable to elicit any tenderness or effusions in any joints. Which of the following is the most likely diagnosis?
A. Rocky Mountain Spotted Fever (RMSF) B. Bone or joint infection C. Kawasaki disease (KD) D. Scarlet fever E. Systemic onset juvenile idiopathic arthritis
C
A. Rocky Mountain Spotted Fever (RMSF) is a tick-borne disease caused by Rickettsia rickettsii. This tick is commonly found in southeastern parts of the U.S., and patients will often come from or have a history of travel to that region. The disease is characterized by headache, fever, myalgia, and a centrally progressing petechial rash originating on the wrists and ankles. The maculopapular rash, and constellation of other symptoms, as well as lack of any recent travel history, makes this diagnosis less likely in this patient.
B. Bone or joint infection should be on the differential given the patient’s refusal to walk, as up to 80% of these infections are in the lower extremities. The fever associated with septic arthritis and osteomyelitis typically are not as elevated as the one presented in this case, and the lack of localized symptoms of warmth and tenderness associated with the lower extremity erythema and swelling make this diagnosis somewhat less likely in this patient.
C. Kawasaki disease (KD) is one of the most common vasculitides of childhood. For diagnosis, in addition to fever of > 5 days, patient must meet four of the following criteria: rash, conjunctivitis, unilateral cervical lymphadenopathy, changes in oral mucosa, or extremity changes (redness/swelling). Our patient does not have lymphadenopathy, but often this is the least common finding in KD. If children have fever with fewer than four of the five clinical findings, they can have “incomplete KD” if they meet certain laboratory criteria.
D. Scarlet fever is caused by erythrogenic toxin produced by Streptococcus pyogenes. Symptoms can include sore throat, fever, “strawberry tongue” and a blanching, erythematous rash with desquamation of the affected areas about six to seven days later as the rash begins to disappear. While our patient does have a “strawberry tongue,” fever, and rash, the description of the rash and other physical exam findings are more consistent with KD than with scarlet fever.
E. Systemic onset juvenile idiopathic arthritis, also known as Still’s disease, is a subset of JIA describing patients with intermittent rash, fever and arthritis. While our patient does present with rash and fever, as well as refusal to walk (potentially a sign of arthritis), systemic onset JIA tends to present with a history of spiking fevers and “salmon” rash occurring when the child is febrile, and disappearing as the fever fades. This is inconsistent with the description of the findings seen in our patient, who does not demonstrate tenderness or effusion in any joint.